Vous êtes sur la page 1sur 30

Vol.

-7, Issue XXV, October-December 2011


ANUNAAD
Faces of Young Nation
Design lessons from
the Steve Jobs' era
IIT-Kanpur
The Quarterly in-house publication of Resonance

Nano Air Purifier

Space Holidays

Nano Medicine

Nano Cosmetics

Nano Computers
Nano Kitchen

Nano Air Bags


Indestructible Fashion

Alternative Power

The Nanoworld
Why life is very different at small scales?
RESONATING THE OF EDUCATION
The Quarterly in-house publication of Resonance

From the Editor


CHAIRMAN
R K Verma The winds of change have been swirling through the world. Be it
EDITOR IN CHIEF the Jasmine Revolution spread across the Arab world, the Euro-
Manoj Sharma zone crisis, the euphoric & rebellious season of Jan Lok Pal Bill,
EDITOR the explosive nature of wiki-leaks breaking down the walls of
Nidhi Agarwal information or even the design God Steve Jobs walking into the
PRODUCTION EDITOR sunset leaving the whole world mourning & tweeting for him
Mahendra Chauhan with his technology marvels under fingers; Change has been the
SYSTEM ADMINISTRATOR eternal constant.
Dharamraj Nagar We at ANUNAAD are also bringing in change, and how! With a
PATRONS MANAGEMENT SUPPORT fresh, coloured look & design, ANUNAAD has rung in the new.
Lokesh Khandelwal Kirti Singh Songara We have been strengthened by the addition of new team
Asheesh Sharma Jitendra Kumar Goyal
Dr. Mrs. V.P. Mittal R. K. Sharma members who would strive to live up to the benchmark
Shishir Mittal S. C. Gupta ANUNAAD has set for itself. We hope that we can carry on the mantle over time and keep
Chandra Shekhar Sharma Arun Shrimali the flag of ANUNAAD flying high.
Ayush Goyal Harish Jain
The cover story brings out the various facet of Nanotechnology, in brief, the
IMPACT EDITORS PRODUCTION SUPPORT
(Academic Content) Manoj Bhatnagar
manifestation of your interest & request. Dealing with things smaller than 100
J.K. Pandey Swati Kripalani nanometres, nanotechnologies are poised to provide fantastically light and strong
Girijesh Dubey Bhawana Gupta materials and revolutionize medicine. From nano-hype to nano-nonsense, the article
Safdar Seraj Shreyas Sharma
S.K. Sinha
sifts sense from speculation. What are nanotechonologies and what might they do for us?
What do we need to worry about? The article sheds light on all this.
DY. MANAGER CIRCULATION SUPPORT
(Corp. Comm.) Pawan Kataria The showcased young Indian leaders' sincere efforts and subsequent achievements have
Vikas Mehta Sanjeev Kumar Godhke aided in bringing a wind of change and pride to the nation. Young Indian Leader Award is
Contact at: 09352890579 Help Desk Staff
(For Advertising Space) Building Incharges
about celebrating the achievements of young Indians who personify the spirit of youth.

PUBLISHERS
Whether or not you are an Apple fan, there's no denying Steve Jobs' prowess as the
Resonance Eduventures Pvt. Ltd. industry's greatest showman. The almost religious atmosphere he could create as he
PRINT MANAGEMENT used to unveil his "magical" products simultaneously thrilled the faithful and irritated
Landmark Printers, New Delhi non-believers. Mr. Jobs was said by an engineer in the early years of Apple to emit a
SUBMISSIONS “reality distortion field”, such were his powers of persuasion. But in the end he conjured
We encourage contributions & submissions. To up a reality of his own, channeling the magic of computing into products that reshaped
submit your contributions & letters please write to entire industries. The man who said in his youth that he wanted to “put a ding in the
The Editor, ANUNAAD, J-2, Jawahar Nagar Main Road,
Kota - 324005 (Rajasthan); or send e-mail to universe” did just that. The article brings out four design lessons of the Steve Job's era.
editor@resonance.ac.in or We remain open to all kinds of article submissions, which will find place of honor
anunaad@resonance.ac.in.
Unsolicited manuscripts will be returned only if depending on the merit of the write-up. On the basis of the feedback received from the
accompanied by a self addressed stamped envelope. readers, we hope to make the magazine even more interesting and informative with each
SUBSCRIPTIONS passing month by bringing in the fusion of varied subjects.
For all subscription-related inquiries please write, We hope that everyone enjoys this edition. Do post your comments on the articles and
email or fax to The Editor, J-2, Jawahar Nagar Main
Raod, Kota - 324005 (Rajasthan) share your feedback through comments, posts as well as direct mails. If you want us to
Tel.: 0744-3192222, 3012222, 2437144, write on a particular subject, do let us know.
Fax: 022-39167222, 0744-2427144
Toll Free No.: 1800 200 2244 Write back to the Editor at editor@resonance.ac.in
e-mail: editor@resonance.ac.in or
Cheers!
anunaad@resonance.ac.in.
READ ONLINE:
On our Website: www.resonance.ac.in
On other Platforms: Nidhi Agarwal
www.blogger.com www.scribd.com, Editor
www.slideshare.net www.wordpress.org
Resonance Eduventures Pvt. Ltd.
HOW TO REACH US By Telephone: CORPORATE OFFICE:
By Mail: 0744-3192222, 3012222, 2437144, Ext.: 355 J-2, Jawahar Nagar Main Raod, Kota - 324005 (Rajasthan)
Letters should be sent to: Tel.: 0744-3192222, 3012222, 2437144,
The Editor, ANUNAAD By Fax: Fax: 022-39167222, 0744-2427144
J-2, Jawahar Nagar Main Raod 022-39167222, 0744-2427144 Toll Free No.: 1800 200 2244
Kota - 324005 (Rajasthan) sms RESO at 56677
They must include the name, address and By sms: Website: www.resonance.ac.in
telephone number of the writer. RESO at 56677
Disclaimer: This issue of ANUNAAD contains advertisements as a service to students. The contents of the advertisements rest solely on the discretion of the
advertiser. Resonance is not responsible for the contents published in the advertisements. Also, the views expressed in articles are the authors' and not
necessarily those of Resonance Eduventures Pvt. Ltd.
In this Issue 3 From the Editor
Content 6
7
Word's Worth
MD's Message:
A Message of Hope
9 News & Views
20 Chemistry Digest
21 Amazing Maths
22 Winners from YCCP
26 Winners from PCCP
33 Career Link: KVPY
40 Olympiad Prowess

Special Article 41 Talent Hunt


45 Think to Win
Nanotechnology 46 Know Mania
49 A fair share of faith
Cover Story 9 Design lessons
52
54
Rainbow
Eureka
Reso Reaper from the 57 Khul Ja Sim Sim
Steve Jobs era
59 Reso Repertoire

IIT-Kanpur 34
Faces of Young Nation 50

48 LK Sir
Reso Seed
Interview
18 38
W ord's
Word's Worth

orth
We can't solve problems by using
A Message of Hope

the same kind of thinking we used


when we created them.
-Issac Newton

Our virtues and our failings are Y ears after years, dreams have persisted, in alleys & lots, in study halls & corridors, in mansions & tenements: I see them floating
around & taking shape everyday, and tens of hundreds of bicycles standing for hours in the parking lot of Institute's campus,
when suddenly become a river, often remind me of how dreams & aspirations need to be reigned in first, and worked upon
before they could be boosted across the nation, else they go tangent. It's important to persistently work towards reigning in your
dreams to convert them to reality.
inseparable, like force and matter. Yes, every person out of corners, pockets & spreads of city, state & country, could rise by education the education system that does not
remind one of the hamster endlessly rotating its hamster wheel with no end in sight, but, by the education that provides adequate
When they separate, man is no more. competition, adequate experimentation, and adequate measurement of efforts & outcomes.
- Nikola Tesla Now, the festive season of Deepawali has taken all of you to meet your dear ones. But the spirit of dedication towards your target
should not get distracted due to this relaxation period. Take your Deepawali holidays in such a spirit that it rejuvenates you & fills in
you a new energy and temperament towards studies. Deepawali, popularly known as the 'Festival of Lights', esoterically means, 'the
awareness of the Inner Light.' We should celebrate Deepawali as the awareness of this Inner Light. This particularly means
outshining all inner darkness, removing all obstacles in the way of our ultimate goal and dispelling all our ignorance

One never notices what towards the most crucial work of our life at this stage i.e., studies.
We all work hard to make the lives delightfully better our lives & of those around us. What's important is how we utilize our time & life

has been done; one can only see on this earth, not how we use it. There is a difference between using the life and utilizing the life. A gold coin with a hole can be used
as a washer and can also be used to buy 1000 washers. The former is 'using the life' and the latter indicates 'utilizing the life.'

what remains to be done. What's important is the energy, efforts and courage we are willing to expend reversing negativism, battling cynicism, suffering and
challenging those who would trample our dreams, taking a stand for magic, and advancing the true potential of our minds & hearts.
- Marie Curie Understand what you need to offer and offer it consistently. Requirement is you have got to go through the furnacing process,
otherwise you'll consume yourself. If the furnacing process is not there, like rot iron you will consume yourself and return back to the
state of iron ore. So, don't be afraid of the hardships. Also, remember that a stone that is afraid to be chiseled can never become a
beautiful statue.
Achievements will come eventually but only tough people would survive the strong currents. Strengthen your concepts &
fundamentals. If you know the raga, you can sing 1000 songs; if you know the song, you will sing only one.
No one can prepare you Today's achievement must not satisfy you completely. We have to start tomorrow with today's benchmarks. We should take
a pledge on this Deepawali to give our cent percent contribution towards studies and to achieve our goal with extra amount of zeal &
for what heights you will energy.
I have great faith in all of you that with self determination and disciplined study schedule, you will meet success.
soar until you spread Happy Deepawali to you and your family!
Best Wishes,
your wings.
- Max Planck R. K. Verma
M.D., Academic Head & Physics Faculty
B. Tech., 1994 (IIT-Madras)
Resonance Eduventures Pvt. Ltd.
ANUNAAD - XXV (October-December 2011) 6 ANUNAAD - XXV (October-December 2011) 7
News & Views
INNOVATIONS
Indian engineer honoured for 'green
steel' technology
shock waves, issuing slight early alerts of
an earthquake," Singhal claimed.
THE NANOWORLD
The project is expected to be of immense
IN THIS ARTICLE
India-born engineering professor Veena
Sahajwalla, whose research led to
help to first responders during a calamity.
The buzz about Why life is very different at small scales

How nanotechnologies may affect our lives


commercialisation of the world's first
“green steel” manufacturing process, was
Pearson Education Services Launches
World Class Product- 'PowerSchool' in balloon.
nanotechnologies reflects both The possible drawbacks of nanotechnologies
honoured with the Nokia Innovation India
Award at the 2011 Telstra NSW Business Pearson Education, India's leading
Zero2infinity's helium-filled aircraft, the
Bloon, is designed to lift passengers to an
the possibilities and the Negotiating the path between lab and
practical applications
Women's Awards. education services provider, today
Sahajwalla, Director at the Centre for announced the launch of PowerSchool, a
altitude of 22 miles where they can cruise
around the earth's upper atmosphere. For
uncertainties of this cutting- What does the future hold for four nano-products?
Sustainable Materials Research and web-based Student Information System perspective, the orbiting space station is
Technology at The University of New (SIS). located 220 miles away, where conditions edge area of science. The big picture

South Wales, is helping the materials A product that has seen wide acceptance (Source: The Wellcome Trust)
are much more ideal to create such an
industries combat environmental of the world over and is an extremely environment. Still, the elevation should
challenges with a technology that reduces successful SIS system for schools will now enable tourists to see the curvature of the
carbon emissions and uses recycled rubber be available in India.The product will help earth and take in panoramic views of other
tyres that would otherwise go to landfills create a collaborative environment for celestial bodies. Essentially, it'll be enough
in electric arc furnace (EAF) steelmaking. School Administrators, Teachers, Parents, for them to get that “Hey! I'm in outer Nano Air Purifier
The process requires less electricity and Air filters will go to the
Students, Counselors and IT Staff of space” feeling, even though in actuality
atomic level to separate Space Holidays
reduces the amount of rubber waste that schools to prepare students for the future. they'll be floating in a region just below clean air from pollutants; Inexpensive fuel
ends up in landfill. The patented use of PowerSchool embraces a wide variety of that known as “near-space.” air quality will return to sources and stronger,
recycled rubber in EAF steelmaking has the functions and supports a wide range of Reaching the destination point will only levels not seen since before easier-to-manufacture
potential to transform production schools and districts - from schools as small take about an hour and passengers will the Industrial Revolution. materials will make
globally. as 25 students to districts as large as have the peace of mind that the journey is Nano Medicine space vacations an
It was her fascination for fiery furnaces and 80,000 students. The deep functions of a emissions-free since the contraption isn't Nanoscopic inexpensive alternative
rugged conditions that convinced PowerSchool allow today's educators to robots will be to earthly destinations.
powered by rocket fuel or other
Sahajwalla to join metallurgical make real-time decisions about each inhaled and
environmentally-unfriendly energy Nano Cosmetics
engineering at the Indian Institute of begin fighting
student while also giving them access to sources. The balloon's dimensions provide Make-up, tattoos and
harmful viruses
Technology, Kanpur. She recalls IITK as one real-time information that allows student a sense of just how much helium is needed colored contacts will
and bacteria
of the most technically challenging places. performance to be addressed in a timely be replaced with
to levitate a pod carrying six people. The before a patient
nanotreatments that
manner. The IT-enabled systems This balloon itself measures 423 feet by 316 even has a sniffle.
alter skin and eye color
IIT-GN teams up with California unanimously accepted student feet, while the passenger housing is for a day, a week or
Institute to develop early quake information system is expected to change roughly 13.7 feet in diameter. even a month.
Nano Computers
warning system the way Indian schools administer work,
After cruising around for a couple of hours, Nanochips smaller
Researchers at the Indian Institute of by not only easing the pressure on Nano Kitchen
the module will begin its fall by releasing than dust particles
Technology, Gandhinagar (IIT-GN) have academia, but also bringing in Machines the size of a modern
helium from the sail end. A parachute will will allow powerful
transparency and control in the way microwave will convert the
undertaken a project to develop a 'slight computers to be
be deployed during the last 30 minutes of cheapest possible materials into
early' earthquake warning system. The records and documents are maintained, carried in one hand.
the home-bound journey. Before landing, any food necessary; in turn,
institute has teamed up with California thereby moving school education to the world hunger and the urge for a
eight vented airbags at the bottom of the
Institute of Technology (CIT), Pasadena, next level. Nano Air Bags chocolate bar will be curbed.
pod will activate to cushion the impact.
USA, to develop a system having dense A net of specialized
Zero2infinity has tested an unmanned Indestructible Fashion
network of low-cost motion sensors Gigantic balloon to lift tourists into molecules will surround an
Enhanced textiles will result
capable of sensing earthquake's early space prototype that reached an altitude of 20 individual and provide
in clothes that resist water,
seismic activity. miles and has completed an initial round invisible protection from
Space agencies like NASA deserve plenty wrinkles and stains while
The goal of the project is to put in place a of venture funding (Series A) and plans to impact and minor accidents.
of credit for taking human exploration providing protection from
network of small devices called put the airship into operation in 2013 at extreme temperatures and
further but, at the same time, they've
accelerometers near an active fault line the earliest. We'll have to wait and see environments without
unintentionally created the false Alternative Power
which can pick up earth vibrations. They over the next few years of this idea ever sacrificing comfort, weight
impression that space travel requires all Nano-engineered solar
gets off the ground. or quality.
can be standalone devices transmitting kinds of elaborate technology like rocket cells will reduce our
data to a central server," said Prof Girish boosters and sophisticated shuttle Manoj Sharma reliance on fossil fuels;
Singhal, project incharge at IIT-GN. equipment. Now a little-known start-up is Vice-President thereby, reducing cost
"A mesh of very low-cost sensors in that poised to show that a trip to the heavens and pollution while
Operation & Business Development protecting our natural
area shall be able to pick up velocity of can be as simple as hitching a ride on a
Resonance Eduventures Pvt. Ltd. resources.

ANUNAAD - XXV (October-December 2011) 8 ANUNAAD - XXV (October-December 2011) 9


...a chicken would be LIFE AT NANOSCALES
as big as the Earth...
The potential of nanotechnology is apparently endless: we are The unusual properties that make the nano-world If you dive into a swimming pool, your inertia will hair on their feet allow them to walk on ceilings
promised everything from the mundane (better paints, self-cleaning so exciting also require keep you moving through the water for several and even to hang upside down from flat sheets
windows) to the bizarre (tiny submarines that will glide through our us to proceed with caution. metres. If you were nano-sized, however, the water of glass.
veins destroying bacteria). would be like treacle its viscosity would soon bring Another difference is that the ratio of surface
Their properties, and their effects on people or the
NANOSCIENCE IS THE SCIENCE you to a gloopy halt. area to volume increases (in a 30 nm particle, 5
environment, may be quite different from those of
Nanoscale objects show markedly different per cent of the atoms are on its surface; in a 3
OF THE EXTREMELY TINY. their macro-forms.
behaviour to large objects. For a nanoparticle in a nm particle, half are). The atoms on the surface
As a result, nanoscience and technologies are attracting considerable
swimming pool, inertia is negligible and viscosity tend to be more reactive than those at the
investment from governments and industry hoping to drive dominates. The water molecules would also centre, so nanoparticle-based materials can be
economic development. So what exactly is nanoscience? bombard the particle because of Brownian motion highly reactive (good for catalysis) or have
And why the excitement? In a nutshell, nanoscience is the science of throwing it around like an aeroplane in constant unusual properties (nano-gold melts at much
the extremely tiny. Nano (from the Greek for 'dwarf') is the prefix for turbulence. lower temperatures than the solid metal).
units of 109. So one nanometre is a billionth of a metre or a millionth At nanoscales, forces that hold surfaces together At nanoscales, the behaviour of individual
of a millimetre. become very strong. For a 'nanobot', this could be a atoms and electrons becomes important, and
The nano size range is usually defined as smaller than 100 nm. But bad thing it would tend to stick to the first surface it interesting quantum effects come into play.
why is nanoscience so special? The key point is that, at nanoscales, met. For geckos, this is extremely useful: nano-forces These fundamentally alter the optical,
materials have strikingly different properties created by the extremely fine electrical and magnetic behaviour of materials.
...a virus would be
Nanoscience is concerned with understanding these effects; as big as a person...
nanotechnologies aim to exploit them to create novel NANOSCIENCE POTENTIAL
structures, devices and systems for a variety of different Nanotechnologies have the potential to touch almost every aspect of our lives.
industries. Because the range of applications is so diverse, it's helpful ...a red blood cell
to think of nanotechnologies in the plural. would be the size C60/fullerenes - In 1996, Sir Harry Kroto, Rick Smalley and Robert Curl won a Nobel
of a football pitch...
Prize for their synthesis of a new form of carbon, C60, which they named
Nanosystems in biology Ironically, the most complex and highly
functional nanoscale materials and machines have already been ...a hair would be as
buckminsterfullerene in honour of Buckminster Fuller, the architect who pioneered
invented by nature. Proteins and other naturally occurring
wide as a river... the geodesic dome. C60 molecules are also called buckyballs.
molecules regulate and control biological systems with incredible In architecture, geodesic domes are known for their strength and lightness. The same is
...a doughnut would be true of buckyballs. When fired at a stainless steel plate at 15 000 mph, they just bounce
precision - from mussel glue, through spider's silk, to water-repelling the size of the UK...
...a flea would off it. And when compressed to 70 per cent of their original size, they become twice as
lotus leaves. be the size
of Brighton... hard as diamond. Their chemistry can also be manipulated. A version in which all of the
Many nanotechnologists are drawing inspiration from biology
carbon atoms are combined with hydrogen (a 'fuzzyball') is more slippery than Teflon
to create new synthetic materials and devices.
just right for coating bowling balls.
So why the worry?
Carbon nanotubes - The discovery of carbon nanotubes in 1991 opened up a new era
in materials science. These incredible molecules have an array of fascinating electronic,
Are nanotechnologies really that new? magnetic and mechanical properties. They are at least 100 times stronger than steel, but
We've used nanotechnologies for centuries. Gold and silver nanoparticles only one-sixth as heavy so nanotube fibres could strengthen just about any material.
centuries we just never knew it. are responsible for some coloured Also, nanotubes can conduct heat and electricity far better than copper, and are already
In this sense, nanotechnology is a pigments, used in stained glass and being used in polymers to control or enhance conductivity, and in antistatic packaging.
'rebranding' of older science, and its ceramics. Computer chips have been LEFT: This coral-grazing parrotfish has particularly Nanoparticles - The term nanoparticles covers a diverse range of chemical and other
influence is evolutionary rather than made using nanotechnologies for the last strong, durable teeth made up of bundles of
nanofibres.
entities. They can be metallic, mineral, polymer-based or a combination of materials.
revolutionary. 20 years, and chemists have been making
RIGHT: Tennis balls made of nano-based They have multiple uses: as catalysts, drug delivery mechanisms, dyes, sunscreens, filters
Old nano polymers large molecules made up of materials remain pressurised for longer. and much more
Nanoparticles are not new: they have nanoscale subunits for decades.
New nano Molecules can also be assembled by
existed widely in the natural world, for Nanowires - Nanowires are extremely narrow threads (less than 50 nm wide). They
chemical synthesis or by self assembly,
millions of years, created Today, there are two approaches to have potential to be used in nanoscale electrical devices. The vision is of electronic
whereby atoms and molecules arrange
by living things or volcanic activity. Nano- manufacturing nanomaterials: 'bottom- chips so small and cheap that they could be used in almost any way. In biology, they
themselves into ordered structures. In
effects are astonishingly common in up' and 'top-down'. could form the heart of extremely sensitive biosensors, identifying molecules
'top-down' approaches, traditional
nature from nonreflective moths' eyes to In the 'bottom-up' approach, associated with disease or the binding of chemicals to a drug target.
engineering techniques such as
extraordinarily efficient nano-lenses in structures are built up atom by atom
machining and etching are used at very Self-assembled nanostructures - If nature can be persuaded to build structures,
crystalline sponges. The enamel of our using sophisticated tools such as the
small scales. manufacturing becomes much easier. Fortunately, self-assembly is widespread in nature
teeth is constructed, in part, by use of scanning tunneling microscope or atomic
Products therefore tend to be refinements (think crystal growth or blood clotting). Many very clever routes are taking advantage of
natural nanotechnology. force microscope. These can pick up, slide
of existing products, Such as electronic self-assembly. This includes use of chemical monomers that naturally polymerise,
Indeed, people have exploited the or drag atoms or molecules around to build
chips with ever more components creating a polymer mesh whose properties can be modified by tweaking the original
properties of nanoparticles for simple nanostructures.
crammed onto them. monomer

ANUNAAD - XXV (October-December 2011) 10 ANUNAAD - XXV (October-December 2011) 11


NANOBOTS: FACT OR FANTASY?
Nanobots were the vision of researcher nanobots just three or four hours to lines, but these are not easy to apply at the THE DOWNSIDE
Eric Drexler. He envisaged tiny robots transform all living things on Earth to grey nanoscale. Many nano-engineers argue that Nanotechnologies offer the potential for huge benefits. But we shouldn't be blinded by dazzling
(hence the term 'nanobots') that could goo. In April 2003, the grey goo scenario we should be looking to chemistry and
nano-promises, say concerned nanosceptics: there may be hidden costs and are the benefits ones we really want?
make nanomaterials, atom by atom. The apparently prompted Prince Charles to biology for inspiration,
nanobots would replicate themselves by enter the nano-debate. He called a taking advantage of Brownian motion, the
Sometimes a piece of technology seems and bind to intracellular molecules. This Civil liberties
taking raw materials, plucking out the meeting of leading 'stickiness' of molecules and molecular
very likely to be mainly good or mainly has been seen in the lab, but it is not clear
atoms they needed one by one, then scientists to discuss nanotechnology. recognition ('lock and key' interactions) to Information, information
bad. Usually, though, especially in an how harmful it is.
assembling a new copy of themselves. A closer look build things. everywhere: will it become even
area as broad as nanotechnology, there Take a deep breath...
Are they really dangerous? Nanobots are nanofiction and likely to Other futuristic prophecies distinguish 'hard' harder to keep personal information to
will be a range of possibilities for use A second possible route of exposure is
He thought so. In his Engines of Creation remain so for decades to come. Drexler nanotechnologies nanofactories,in which ourselves?
and abuse, a mix of good and bad effects. through the lungs, if people breathe in
(1986), which introduced himself has disowned the grey products are built by mechanical processes These days everyone, from governments
Green or black? nanoparticles present in the atmosphere.
nanotechnology to the public, Drexler goo scenario, and Prince Charles has and 'soft' nanotechnologies, those based on and health workers to banks and
Again, very little is known about how
suggested that if a few nanobots were to acknowledged it is not an issue. biological systems. Despite the potential for nano-clean- supermarkets, seems to want more
nanoparticles behave in our lungs, but it is
multiply out of control, they could form a The practical challenges would be Soft nanotechnologies merge into up, one of the biggest fears is the personal information. One set of
clear that they can be taken up by cells in
swarm of tiny, precisely engineered yet immense, and there seems little need for biotechnology. They can be seen in projects possibility of environmental damage. the lung, triggering inflammation. They applications of nanotechnologies will
lethal machines that together pull apart self-replication anyway. Even physics is that combine biological and physical Given their small size, the worry is that can also get into the bloodstream, and offer lots of possibilities for collecting
every living thing in their path, atom by against the idea. Nanosubs, inside blood systems, and those that are attempting to nanoparticles will easily become new data. Tiny sensors, embedded in
atom. Japanese scientists have used

F
create 'minimal viable cells', building up clothes, products or even bodies,
He imagined that this rampaging swarm
vessels, would be smashed to bits in the
nanoworld simple cells from scratch. As biological
airborne and spread through the
atmosphere, or will contaminate
AST nanoneedles 100 nm wide to could monitor the movement of
might look like 'grey goo'. According to We als o ten d to th in k ab o ut systems are known for their powers of self-
replication, this has led to fears that the 'grey
aquatic environments. Once in the ACT deliver materials to very
specific points in the cell.
people or products, or record health
information. It might mean we are
one estimate, it would take replicating 'manufacturing' in terms of production
goo' may in fact be a 'green goo'. observed, sorted,
transport of nanoparticles through

F
Researchers in Florida, USA,
AST have devised a nanoparticle- nerve tissue to the brain has also profiled and classified wherever we
go, and whatever we do.
been seen.
HOW THEY ARE USED ACT based bioassay that can detect a
single bacterium within 20 minutes.
Carbon nanotubes may be an especial
This can be harmless, if it helps service
providers or companies give us what
cause for concern, as they resemble
Medical applications nanoparticles, but they could also play an various biosensors to monitor a soldier's environment, they could accumulate in we need. But it may also make it harder to
asbestos fibres, which caused cancers in
The medical potential of important role in protecting the health, and nanosized silicon carbide living organisms, as many harmful protect privacy, or keep personal
workers who breathed an asbestos-
nanotechnologies is huge. Already on the environment. A typical application would particles for physical protection. The substances do, or damage ecosystems. information confidential.
laden atmosphere. As with
market in the USA are wound dressings be based on a column containing nano-battlesuit is being developed at Nanoparticles and nanotubes may nanoparticles, new research is These concerns are not new, and are
that exploit the antimicrobial properties of nanoparticles that bound to a particular Massachusetts Institute of be harmful. But we don't really know. needed on their possible effects, but already discussed in relation to barcodes,
nanocrystalline silver. Nanomaterials contaminant. As water passed through the Technology's Institute for Soldier ID cards, computer databases and CCTV
Nanotechnologies are partly about again early signs are that they might pose a
could make good implants. column, the contaminant would be Nanotechnologies. cameras. So nanotechnologies may not
crafting new materials, and these may threat to health.
absorbed onto the nanoparticles. The Cosmetics give rise to any new issues, but they are
Nanoparticles such as nanocrystalline need safety testing. But are there new There will also be puzzles to solve in
nanoparticles could then be retrieved (e.g. likely to intensify existing debates.
z i rc o n i u m ox i d e ( z i rc o n i a ) a n d Nano-titanium dioxide and zinc oxide can hazards arising from the kinds of material measuring and monitoring.
nanocrystalline silicon carbide are strong, by removing them absorb and reflect UV light, while also on offer? Nanoparticles are often too small to show
lightweight, resistant to wear and magnetically) and the contaminant being transparent to visible light. They are Who benefits?
UNFORTUNATELY, THERE IS A LOT OF up with standard instruments.
corrosion and (unlike many other washed out. already used in sunscreens. The SPECULATION BUT LITTLE HARD Will nanotechnologies lead to a nano-
nanoparticles) inert. Military applications cosmetics industry has invested INFORMATION. divide between rich and poor?
For the future, there is potential for New classes of nanopolymers are being heavily in nanotechnology. New New drugs or
The main concern is about nanoparticles. new toxins? Many people fear that nanotechnologies
nanoparticles to be used as vehicles for developed that can be sprayed on to a products are claimed to penetrate deeper Because their properties differ from larger will further increase the gap between the
gene and drug delivery. soldier, to form a suit without seams. The into the skin or to have other benefits. For forms of the same substance nano-gold, 'haves' and the 'have nots'. Certainly,
Environmental applications fabric is planned to contain embedded example, cosmetics that slowly release for example, is not like solid gold there early examples of nanoproducts have
There are environmental concerns about enzymes that detect and break down vitamins are in development. are fears that existing safety measures been driven by a rich-world agenda:
chemical and biological warfare agents, may not be adequate. suncreams, tennis balls, tennis racquets,
Some forms of nanoparticle appear to be laptop computers and so on.
able to pass through the skin.This has its But it is not inevitable that
advantages for example, it is a way to get nanotechnologies will heighten global
vitamins into the body using cosmetics. inequalities. Applied the right way, they
But, given their size, they could interfere

F
could provide many benefits (e.g.
AST A team of German and Swiss scientists has
calculated that nano-forces allow spiders to carry
with the function of proteins on the renewable energy from solar power,

ACT more than 170 times their own body weight. surface of cells, or be taken up into cells medical diagnostic kits, cheap water

ANUNAAD - XXV (October-December 2011) 12 ANUNAAD - XXV (October-December 2011) 13


TRICKY INNOVATION active opposition to certain products. Environment... products, new methods can assess costs
Sometimes it is simply successful over the whole life-cycle, not just
The road from idea to genuinely new product to market a In the past, environmental impacts were
marketing that makes all the difference. manufacture. So the cost of a nuclear
different type of risky business. Many rarely thought about. Environmental
application is long and products fail the test of consumer Public attitudes to nanoscience appear
harms are easy to pass on to someone else
power station will eventually include
fairly positive at the moment. But this is paying to dismantle it and store the waste.
rocky many factors will demand. air and water often shift pollutants far
But whether all relevant costs (and
probably a poor predictor of future
On the other hand, particular demands away from the source. And they're often
influence whether a responses. Few people have heard much nonfinancial social impacts) get factored
can shape the technology on offer. 'invisible' until fish start dying or winter
Safer lives or product ever sees the light about nanotechnology, so this positive in is open to debate.
Personal computers have astonishingly disappears. Environmental impact
more surveillance? outlook may derive from a general feeling
of day. good graphics because that is what game-
that science and technology do more good
assessment is now routine for big projects
Science and technology offer a huge players want, even though the mass of like dams, factories or airports. For ucts,
purification or waste clean-up). Nano- than harm.
range of possibilities for feeding, email and word-processor users may
based technologies may also allow
clothing, mending, defending or just never see them in action.
countries to 'leapfrog' outdated
amusing ourselves. Only some ever Because of the way that health products
WHERE NOW FOR NANO?
technologies.
become part of our lives, however. This is are used, the situation here is slightly Nanotechnologies are finding a huge range of applications, as these case studies
As usual, the issue depends on the partly because time and money are different. Consumer demand plays less of illustrate. How do you think they will be affected by the factors discussed in this issue?
priority we wish to give to problems always limited. If you spend all your cash a role and there is a growing trend to Which would you back for success?
affecting the developing world. There on heart transplants, for instance, then assess cost-effectiveness of new
has been little sign so far that the needs of cholesterol testing to prevent heart treatments.
poor countries are shaping the nano- disease may lose out. LIGHT FANTASTIC
But economic realities still dictate what
revolution. But there are a host of other things that happens. Pharmaceutical companies Quantum dots… altering the size of the core, researchers can each heartbeat.
An alternative is for countries to seize the influence technological futures, such as
constantly search for blockbuster drugs Just a few thousands of atoms each, fine-tune quantum dots to emit light at a Adding antibodies or other molecules to
initiative for themselves. politics, economics, and social,
ideally tackling a widespread problem for quantum dots are being used as tiny variety of wavelengths, producing a set of the dots can be used to target them very
And some emerging economies, such as environmental and technological issues.
many years. Less common problems, or beacons or markers that can be used to multicoloured markers. The dots can be a specifically. Whole cells can be labelled
China and Mexico, have made moves to Politics... those affecting the developing world, watch and track cells, genes, proteins and thousand times brighter and last much and tracked more than 100 different cells
develop local nano-capacity. hardly get a look in.
Politicians in democratic countries other small molecules. When a beam of longer than conventional dyes. In one simultaneously. Proteins on the surface of
But there is always a fear that new Financial opportunities can also create study, scientists used quantum dots to
promise to do things on our behalf, such light is shone upon a quantum dot, the cells, such as cancer cells, can be
technologies can be too seductive, being incentives for companies to develop watch blood flow in the tissues of living
as manage the economy, improve public electrons in its core become excited and identified. Even the movements of
pursued more for their own interest than medical products of dubious mice. The images were so detailed they
health and maintain effective defence. re-emit light at a wavelength that individual proteins can be followed inside
because they offer the best prospect for value, to be marketed at the general showed blood vessel walls rippling with
And they pay attention to safety issues, depends on the size of the core. By a cell.
solving existing problems. Many population.
and must heed public opinion not least
(probably most) of the world's biggest
because they want to get re-elected.
problems could be tackled with tools Technological... DETECTIVE AGENCIES
When it comes to technology, the
we already possess. Another problem for the would-be Diagnostics… nanoshells are like tiny Maltesers, but have Nanoparticles are being developed that
economy tends to come first. Science is
entrepreneur is knowing when the time is a crunchy core of glass and an outer case of recognise proteins produced only by
now increasingly supported not because A highly promising use of
ripe to invest in a particular technology.
RISKY BUSINESS new knowledge is valued in itself but
There is a big gap between the visions of
nanotechnologies is in diagnosis of gold rather than chocolate; unlike cancer cells. A longer-term plan is to
because it is a good 'investment'. disease. The key principles are the specific quantum dots, which re-emit light energy, produce a cocktail of different coloured
As well as having many We are now supposed to be living in a
nano-manufacture and what is now
recognition of a molecule linked to a gold nanoshells absorb or scatter the quantum dots to help doctors spot early
possible in the lab. Even if something energy. Quick blood tests are being indications of cancer, or identify different
b e n e f i t s , n e w 'knowledge economy' - so we need to works in the lab, it may take much more
disease state and detection of this
recognition. Nanotechnologies offer the developed using nanoshells coupled to types of tumour. A technique called bio-
nanotechnologies could concentrate on high-tech industries and work to scale it up for wider use. The
prospect of very sensitive recognition and molecules that detect diseaseassociated barcode amplification has been used to
jobs requiring special skills education w h o l e p ro c e s s o f t e c h n o l o g i c a l
backfire. How do we tackle and a strong research capability. very quick detection. Early-warning test proteins. Changes to the nanoshells' identify tiny amounts of a protein that
development can take decades, and it is optical properties when they bind to the may be an early hallmark of Alzheimer's
risks and the unknown? Governments try to imagine future often evolutionary rather than
kits for disease are being developed using
quantum dots and gold nanoshells. Gold target can easily be detected. disease.
Technologies have improved our lives technological possibilities, to help guide revolutionary. There have been bold
immeasurably, but we now know that spending choices. predictions about gene therapy, but
they can bite back. From cars and hardly any patients have seen the benefit GREAT STRIDES FORWARD
chemicals to medicines and microchips,
new things bring costs as well as benefits. Economics... yet mostly because it has proven so Nanopants… the textile surface, presenting a waterand
technologically demanding. Nanopants (or nanotrousers) are stain-resistant surface to the outside world.
But how do we balance potential gains In the end, someone has to pay for all this
stuff. By and large it is left to the garments whose fabric has been treated If you happen to spill coffee or orange juice
against possible drawbacks? This is a
with a product containing polymer chains on your 'nanopants' the liquid simply
complex situation that now involves commercial sector to develop products. Social influences... beads off and falls harmlessly to the floor,
governments, industry, universities, So what actually gets made depends on to improve their resistance to staining.
Consumers make or break a new product. rather than leaving a stubborn stain.
consumer and campaign groups and what can be sold. Quite simply, hydrophobic bits of the
But social influence can go further, from
ordinary people. All this makes innovation bringing a chain will arrange themselves away from
creating fashions that increase demand to

ANUNAAD - XXV (October-December 2011) 14 ANUNAAD - XXV (October-December 2011) 15


A Japanese company has
BIONANO OR NANOBIO?
Molecular machines…
Nanotechnology may be a new human
the enzyme ATP synthase, which makes
ATP, the chemical energy source for nearly
F AST developed 10 nm-diameter
particles that coat pollen and
ACT stop it releasing hayfever-
causing allergens.

all living organisms, is actually a tiny very robust. Others are combining
activity, but nature has been at it for
rotary motor. It has been attached to a components from different organisms,
millions of years. Every cell contains
nanoscale bar magnet made of nickel: this such as the light-detection system from
thousands, even millions, of machines
hybrid device could be a potential plants and energy-generating enzymes
and factories that can build remarkable
nanomolecular motor of the future, from mitochondria, to create biologically
structures with an efficiency that today's
although the biological component is not based solar cells.
scientists can only marvel at. For example,

NANOSCIENCE:
THE BIG PICTURE
m Nanoscience is the science of the extremely small
objects smaller than 100 nanometres (0.00001 cm).
m At these scales, the properties of materials change
dramatically. Factors such as Brownian motion,
surface stickiness and quantum effects become
important.
m Nanotechnologies are based on a range of new
materials, including carbon C60, carbon
nanotubes, nanoparticles, nanowires, and polymers
based on nano-size subunits.
m A huge range of applications are possible, based
on stronger, lighter or smaller materials, or
compounds with unusual optical or electrical
properties.
m Early applications are enhancing existing products
- tennis racquets, golf clubs, sunscreens.
m Possible medical applications include better
implants, wound dressings, diagnostics and cancer
treatments.
m Combining biological molecules with
nanomechanical components is creating radically m Environmental concerns focus mainly on nanoparticles but
new materials; these are at an early stage of very little is known about their impact on living things.
development. m Self-replicating nanobots are extremely unlikely.
m The public currently has little input into policy m Nanotechnologies could increase the divide between rich and
making in science and technology. poor, but could also provide products useful to the developing
world and may be easier for poorer countries to take up.
m Several groups advocate greater public involvement. Others
doubt it is feasible (or even desirable).

ANUNAAD - XXV (October-December 2011) 16


A
RE O
bhishek Katyal, Reso Alumnus, now working at Microsoft Research
India, Bangalore, attempts to resolve academic concerns of students
preparing for IIT-JEE and other prestigious competitive exams.

EED Mr. Abhishek Katyal


Year of enrollment at Resonance: 2005-06
Year of leaving Resonance: 2006-07
Reso Roll Number: 501752
Branch at IIT: CSE
Year of enrollment at IIT: 2007
Year of leaving IIT: 2011
Company Name: Microsoft Research India, Bangalore
IIT: IIT-DELHI Date you started working here: 2011

One can choose to remain focused on cricket, it will be difficult for you to stop Do's- but even then, do not let out expressions
goals & one can fall for sources of yourself from joining them. If your friends 1) The 11th class is more difficult than 12th like “I can't do anything. I wish I had a
distractions. Would you like to suggest are studying, you will also get motivated to class in Kota as you have to learn a lot of brain like that of a topper.”
some do's & don'ts for students who study. It is as simple as that. things. So, try to find a good senior in your Always remember everyone has his
are taking coaching in Kota? 3) Always remember, “If you are lured to PG/hostel or your city who can guide you. natural pace of understanding. Everybody
Don'ts- do it for the first time, you will definitely Ask him about how to attempt the exams, has some strengths and weaknesses. If you
1) Never go to gaming and movie shops as be lured to do it for second time as well”. which books to follow, how to plan your can identify your weaknesses and
these create addiction. Keep in mind that if So, never think, “Let's leave home-work studies etc. strengths, you can improve a lot. Try to
you clear JEE, you will get a lot of movies today & watch movie Just for today. We'll 2) A lot of students lose their confidence improve upon your weaknesses slowly
to watch and lots of gaming to do. not do it in future.” Remember - If you do when they are unable to score well in tests and gradually, and utilize your strengths.
2) Always try to have a good friend circle. it once, you will definitely do it the second despite their best efforts being put in, and
If your friends are going for a game of time as well. it is quite natural. I know it feels really bad

Lacs of students join coaching How can one gain confidence & IIT-JEE Physics is all about getting your Most of the syllabus is covered while 3) Try to find time for some physical make IIT-JEE paper are not from a different
institutes every year. How best a develop interest in subjects? concepts right and then practicing those preparing for IIT-JEE. Before the exams, it activity like jogging, skipping etc either in world. The paper of IIT-JEE is going to be
student can take the advantage of Before you read answers to the other for gaining speed. Whenever you are is good to go through the last 10 year the morning or the evening. Do it from the same syllabus as the institute's
coaching in a good coaching institute? questions, I would strongly recommend completely stuck on one particular kind of papers for boards. whenever you feel mentally tired. periodic test. Of course there will be some
I consider myself really lucky that I got the you not to read all the questions in one go. problem, the probable reasons are: Never compromise on sleep and food, and variations like the level of difficulty or the
opportunity to study for 2 years in Kota Read 2-3 questions on daily basis, think a) You do not know which concept to Many students who are preparing for be regular. It will help you remain length of the test. It can be slightly easier
and I thank God and my parents for it. over them and discuss those with your apply. competitive exams remain curious energetic and work efficiently. or difficult than the periodic test but if you
Some of you might not be able to realize it friends during your free time. Only then, b) You know which concept to apply but about the plan of studies & daily are performing well in the periodic tests,
today but I am 100% sure that you will you will be able to draw a meaningful do not know how to apply. routine of successful students. Any Any useful tips to learn formulae you will definitely do well in IIT-JEE.
definitely realize it at some point in your conclusion out of each. Also, my thought c) The problem is a mixture of 2-3 advice for them to have maximum effectively and remember them for
future. process and way of studying might not be concepts and you do not know how to productivity & output? long? How can one deal with the emotional
If you want to clear IIT-JEE, there cannot be the same as yours. So, do not worry if you merge all those concepts. Revising your class notes and completing Most of the formulae are remembered as & mental stress usually associated with
any place better than Kota. Whenever you are not able to do the things as I have So, what should you do? the home work should be your first you practice. There are some formulae IIT-JEE preparation?
feel down or depressed, think about your suggested. In case (a), open your class notes and look priority. I followed some of the following that are less used. Instead of just Be patient. I know it is really frustrating
classmates who could not come to Kota, The first and the foremost thing is to pace through all the concepts that have been techniques to maximize productivity: cramming those formulae, try breaking when you work very hard and even then
who do not have good teachers, who do with the class. Always make sure that you taught in the class and go back to the 1) Take 5-10 min break between 50-60 them into parts and understand each part. you do not get good marks in the test due
not have good study material and have revise your class notes and the homework problem again. minutes of studies. The purpose of the For e.g., the formula for electric field by a to some silly mistakes. Take it positively
nobody to guide them. This will give you discussion within 1-2 days of discussion. If In case (b), write down the points you are break is to relax and refresh your mind. charge carrying wire can be remembered that you did not do those mistakes in IIT-
energy and motivation to perform better. you revise them later, there is a possibility facing problems in. When you will get to Walking in the corridor, taking a small nap, as: JEE and still have the chance to improve
You can learn a lot from your friends apart that you will forget many of the things told know the solution to the problem, you will listening to some soothing music etc are a)Constant factor: Like 2*K* lambda upon them. Analyze your mistakes and
from your teachers. Remember the golden by the teacher. Also, it is a well proven fact remember the concept for a long time. some of the activities. Do not watch TV or b)Proportionality factor: Now remember you will definitely improve.
rule that “Knowledge increases by that revising in this manner helps retain Case (c) will not arise if you have play video games as instead of relaxing that electric field by a point charge is Also, keep in mind that when you are
sharing.” If you are not able to solve a the notes & discussions for a longer period sufficiently dealt with case (a) & (b) your mind, it will stress you even more. proportional to 1/r2 and by line charge it is giving your 100%, others are also giving
problem and your friend is able to, ask him of time. 2) Do not study a subject for too long. 1/r. their 100%. So, it is really foolish to get
how he/she approached the problem, A lot of students are caught between Keep changing the subjects. Studying frustrated when you do not get a good
what was his/her thought process. I know What, according to you, a student the preparation of competitive exams theory continuously can be boring as well. Performance in institute's periodic test rank. Also keep in mind that some people
you get very little time for collaborative should do when stuck on any problem and the Board Exams. How can one So, I used to take breaks between practices & performance IIT-JEE How are these are naturally good at problem solving and
problem-solving, as the homework is so in a particular subject (Take Physics, for balance IIT-JEE syllabus & School of numerical and study those things which two things inter-connected? it is very difficult to defeat natural talent
much but these things can be done during example)? What could be the reasons? syllabus without being chaotic? need to be memorized. For example - Always keep in mind that the people who with hard work.
lunch/dinner time or on Sundays. formulae, inorganic chemistry.

ANUNAAD - XXV (October-December 2011) 18 ANUNAAD - XXV (October-December 2011) 19


CHEMISTRY
DIGEST
O CH3 CH3 1.2 Cauchy-Schwartz

H
CH3
CH2– C
( ) (b21+b22 +...... +b2n) ³ (a b + a b
Let a1, a2, ......... an, b1, b2,........ bn be nonzero real numbers. Then a21 +a22 + ...... +a2n

cis-2-butene
Revise the basic of organic reactions Succinic Br H Br2

C=C
1 1 2 2
O H Br
CH– anhydride + ........ + an bn)2 (*)
2 C H Br
O Br H a a a
with equality if and only if 1 = 2 = ...... = n

CH3
–HO CH3 b1 b2 bn

H
2  CH3

dl–2,3-dibromobutane This is the well-known Cauchy-Schwartz inequality. The standard elementary proof uses the properties of the quadratic
CH2 – CH2
CH–
2 CH 2 HO/H
2
+
function: consider the function
CH2– CH2 CH3
NH/3  COOH COOH H2O[O] f(x) = (a1 - b1X)2 + (a2 - b2X)2 + .......... + (an - bnX)2
C C O H OH
N

Electrolysis
Clearly, f(x) ³ 0 for all real x, therefore, being a quadratic function, its discriminant  must be negative or zero. The inequality

Of its K-salt
O CN CN
H OH Baeyer's Reagent
H
Succinimide
KCN CH3
1 (
follows by observing that  = 4 (a1b1 + a2b2 + ......... + anbn)2 -4 a2 +a2 + ...... +a2
2 n ) (b21+b22 +...... +b2n)
Meso-butane

H 2/Lindlar's
Br2 2,3 -diol If we have equality in (*), then  = 0 and the equation f(x) = 0 has a real root x0. But then

Catalyst
KCN alc.KOH
CH3–CH2–CN CH3– CH2 – Cl CH2= CH 2 CH2– CH2
a a a
SnCl2/ HCl
CH3– CH2 – CH = O
Br Br (a1 - b1x0)2 + (a2 -b2x0)2 + ............ + (an - bn x0)2 = f(x0) = 0, Þ 1 = 2 =...... = n = x0
b1 b2 bn

O/
4

H2O/H+ then the equation f(x) = 0 has a real root, so  ³ 0. Since  cannot be positive, it follows that  = 0 and we have equality in (*).
HS

CH3 – CH2COOH
2

(1) alc.KOH
(2) NaNH 2
nc .

Another proof uses a simple lemma which can also be helpful in proving a large number of algebraic inequalities.
(I) O3
Co

liq NH3/D 2
SOCl2

2 2
aq.KOH (II) Zn – H2O Lemma. If a, b, x, y are real numbers and x, y > 0, then the following inequality holds a + b ³ (a+b)
O x y x+y
(I)CH3– Mg I
CH3– CH2OH CH2= O
CH3 – CH2 – C – NH2 (II) H2O/H
+ (I) NaNH2(1ev)
CH3–Cº C– H
(1) NaNH2
CH3 – C º C – CH3
The proof is straight forward. Clearing out denominators yields a2y (x + y) + b2 x(x +y) ³ (a+b)2 xy,
(II) CH3I (2) CH3I which readily simplifies to the obvious (ay - bx)2 ³ 0. We see that the equality holds if and only if ay = bx, that is, if a = b
2
O

Br2/KOH x y
HN

++
Hg / H2SO4 Hg2+
Cu/ a21 a22 a2n 2
CH3 – CH = O
Þ ..... ³ (a1+a2+......an ) for all real numbers a1, a2,.........,an and x1, x2,...........xn > 0, with equality

Li/NH 3(l)
x1 + x2 + + xn

Na/NH 3(l)
CH3– CH2 – NH2 –H2 Cu/
(i)CH3–MgI CH3– CH – CH3 CH–
3 C – CH 3
x1+x2+.......+xn
– H2

or
b  +
(A) (ii)H2O/H OH a a a
CH–CH=CH–COOH CH3 CH3 (i)CH3–MgI O if and only if 1 + 2 =........ = n Let’s see our lemma at work!
3
x1 x2 xn
OH/

dryHCl Cu/ CH – C –CH (ii)H2O/H


+
CH–CH–O–CH–CH
o

3 3
CH3– C = CH2 – H2O 3 3
a b c 3
OH CH3 Problems 1. Let a, b, c be positive real numbers. Prove that + + ³
O O b+c c+a a+b 2

CH 3
CH3 –CH = CH – CH = O

trans-2-butene
H
CH–CH–O–CH–CH
3 3 H Br Br2
Crotonaldehydle

Metaldehyde
Solution. Observe that , a b c a2 b2 c2 (a+b+c)2

C=C
H Br + + = + + ³
b+c c+a a+b ab+ac bc+ba ca+cb 2(ab+bc+ca)
CH3
CH 3 (a+b+c)2
H
Meso-2,3-dibromobutane 3
so it suffices to prove ³
CH3 CH3 2(ab+bc+ca) 2
H2O[O]
A short computation shows that this is equivalent to a2 + b2 + c2 ³ ab+bc+ca, which yields (a-b) + (b-c) + (c-a) ³ 0
2 2 2
H OH HO H
(A) = (CH3CO)2O/CH3COONa Baeyer's reagent Problem 2. 4 4
Let a and b be positive real numbers. Prove that 8 (a + b ) ³ (a+b)
4
2
HO H H OH (a+b)2
4 b4 2 2 4
a (a +b ) 2 = (a+b)
CH3 CH3 Solution. We apply the lemma twice. a4 + b4 = + ³ ³
1 1 2 2 2
dl-butane-2,3-diol
Try to use the lemma to prove the following inequalities.

2 2 2 9
1. Let x, y, z > 0. Prove that + + ³
x+y y+z z+x x+y+z
x y z 3
2. Let a, b, x, y, z be positive real numbers. Prove that + + ³
x+y y+z z+x a+b
a2+b2 b2+c 2 a2+c 2
3. Let a, b, c > 0. Prove that + + ³ a+b+c
a+b b+c a+c
1 1 1 3
Girijesh Dubey (GD Sir) 4. Let a,b, c be positive numbers such that abc = 1. Prove that 3 + 3 + 3 ³
a (b+c) b (a+c) c (a+b) 2 Safdar Seraj (SS Sir)
Sr. Reso Faculty Member, Chemistry
HOD - Maths, Patna Centre

ANUNAAD - XXV (October-December 2011) 20 ANUNAAD - XXV (October-December 2011) 21


Performance Matters VIPUL-BC (Target IIT-JEE 2013) Cum.% up to PT-2
Winners From
Resonance Yearlong Classroom Contact Programmes (YCCPs) -IIT-JEE Division
Rank - 1

VIKAAS-A (Target IIT-JEE 2013) Cum.% up to CT-2


Rank - 1 Rank - 2 Rank - 3 Rank - 4 Rank - 5

Tanmay Mothe
Batch: MBGOJBBG1, SC: Mumbai - Govandi
Cumm%: 61.51
VISHWAAS-F (Target IIT-JEE 2012) Cum.% up to CT-3
SHUDHATMA JAIN PALAK JAIN PARTH SHAH GARVIT JAIN YASH GUPTA Rank - 1 Rank - 2 Rank - 3 Rank - 4
Batch: KTJAIA Batch: UDAJAAI Batch: JPJAA1 Batch: KTJAIA Batch: UDAJAAI
SC: Kota SC: Udaipur SC: Jaipur Central SC: Kota SC: Udaipur
Cumm%: 93.51 Cumm%: 90.67 Cumm%: 86.43 Cumm%: 86 Cumm%: 82.48

Rank Name Batch SC Cum.% Rank Name Batch SC Cum.%


6 Vipul Bachani KTJAIA Kota 82.22 23 Kaushal Jha KTJAIA Kota 75.13
7 Ayush Agarwal KTJAIA Kota 81.05 24 Sanchit Dayal KTJAIA Kota 74.87
8 Shubham Mittal KTJAIA Kota 80.49 25 Saksham Gupta KTJAIA Kota 74.64
9 Sheel Nidhan KTJAIA Kota 80.4 26 Kriti Joshi UDAJAAI Udaipur 73.97
10 Sudhir Kumar KTJAIA Kota 79.54 27 Neel Kasat KTJAIA Kota 73.14
11 Bhanu Chaturvedi KTJAIA Kota 79.44 28 Deepanker Mishra KTJAIA Kota 73.03
12 Deepak Goyadi KTJAIA Kota 79.32 29 Akshay Nagpure NPAA1 Nagpur 72.15 DEEPENDRA PATEL HEMANT YADAV ASTEEK CHOUHAN PRATIKSHA
13 Raj Maheshwari KTJAIA Kota 79.18 30 Nishant Gupta KTJAIA Kota 72.11 Batch: KTJFF1, SC: Kota Batch: KTJFFH1, SC: Kota Batch: KTJFF1, SC: Kota Batch: CKTJFF1, SC: Kota
14 Sarthak Vijay KTJAIA Kota 78.96 31 Ankit Saran KTJAIA Kota 71.99 Cumm%: 75.62 Cumm%: 61 Cumm%: 60.37 Cumm%: 60.35
15 Aishwary Gagrani KTJAIA Kota 78.89 32 Rituj Jain JPJAA1 Jaipur Central 71.91
16 Rishabh Raj KTJAIA Kota 78.38 33 Sidharth Tiwari JPJAA1 Jaipur Central 71.35
VIJETA-P (Target IIT-JEE 2012) Cum.% up to CT-3
17 Ayush Shringi KTJAIA Kota 77.71 34 Archit Chaudhary KTJAA1 Kota 70.94 Rank - 1 Rank - 2 Rank - 3 Rank - 4 Rank - 5
18 Yash Mittal KTJAIA Kota 76.69 35 Naman Sogani KTJAIA Kota 70.85
19 Ishit Murdia UDAJAAI Udaipur 76.48 36 Shubham Barai NPAA1 Nagpur 70.53
20 Anirudh Sharma UDAJAAI Udaipur 76.43 37 Iqra Altaf KTJAIA Kota 70.38
21 Deepam Jain KTJAIA Kota 76.12 38 Yajuvendra Singh KTJAIA Kota 70.26
22 Akshay Sinha KTJAIA Kota 75.58 39 Ishu Choudhary KTJAA1 Kota 70.01

VIPUL-B (Target IIT-JEE 2013) Cum.% up to CT-2


Rank - 1 Rank - 2 Rank - 3 Rank - 4 Rank - 5

VIKAS GARG NIMIT KUMAR SINGH ROHIT KUMAR GUPTA SIDDHARTH PATEL HARSHIT AGARWAL
Batch: KTJPIP Batch: JPJPP1 Batch: KTJPIP Batch: KTJPIP Batch: KTJPIP
SC: Kota SC: Jaipur Central SC: Kota SC: Kota SC: Kota
Cumm%: 81.21 Cumm%: 80.60 Cumm%: 77.95 Cumm%: 77.78 Cumm%: 77.20
Rank Name Batch SC Cum.% Rank Name Batch SC Cum.%
6 Himanshu Goyal KTJPIP Kota 75.65 14 Rohit Pruthi DLNWJPW-2 Delhi-North-West72.49
7 Shubham Jain KTJPIP Kota 74.76 15 Vaibhav Kewlani KTJPIP Kota 72.08
8 Tushar Marda KTJPIP Kota 74.02 16 Vikas Meena KTJPIP Kota 72
9 Pankaj Goyal KTJPIP Kota 73.59 17 Ashutosh Mittal KTJPIP Kota 71.17
K. JITENDRA LAXMAN JANGLEY SIMRAN KAUR BINDRA NIKHIL BANSAL MAITHILI PATEL 10 Ankit Kumar Sinha KTJPIP Kota 73.43 18 Parshant Chawla KTJPIP Kota 70.44
Batch: KTJBIB Batch: KTJBIB Batch: DLNWJB1 Batch: Batch: ABJBB1 11 Rajat Jain KTJPIP Kota 72.87 19 Krishan Mittal JPJPP1 Jaipur Central 70.4
SC: Kota SC: Kota SC: Delhi - North-West SC: Jaipur Central SC: Ahmedabad 12 Himangi SaraogiMBGOJPPSRP1Mumbai - Govandi 72.65 20 Astuti Sharma KTJPIP Kota 70.07
Cumm%: 89.11 Cumm%: 86.25 Cumm%: 73.47 Cumm%: 71.91 Cumm%: 71.48 13 Tanushree Gupta KTJPIP Kota 72.63

ANUNAAD - XXV (October-December 2011) 22 ANUNAAD - XXV (October-December 2011) 23


Winners From Performance Matters
VIJAY- R (Target IIT-JEE 2012) Cum.% up to CT-2 Resonance Yearlong Classroom Contact Programmes (YCCPs) - AIPMT Division
Rank - 1 Rank - 2 Rank - 3 Rank - 4 Rank - 5 SANKALP- MP (Target AIPMT 2012)
SAKSHAM- MA (Target AIPMT 2013) Cum.% up to CT-1
Rank - 1 Rank - 2 Rank - 3 Rank - 4 Cum.% up to CT-2
Rank - 1 Rank - 2

KARTIKEY SAINI VASEEM AEHMAD JAYA RANI OJHA DRUPAD DWIVEDI


Batch: UDAMAA1 Batch: UDAMAA1 Batch: KTMAA1 Batch: KTMAA1 SHIRISH SINGH SHAKTI KUMAR
AJAY SHARMA DHEERAJ AGARWAL ASHUTOSH DOLIYA MOHIT AGARWAL ISHA GOYAL SC: Udaipur SC: Udaipur SC: Kota SC: Kota Batch: KTMPP1 Batch: KTMPP1
Batch: KTJRRJ1 Batch: Batch: KTJRRU1 Batch: KTJRRJ1 Batch: KTJRRJ1 Cumm%: 74 Cumm%: 71.47 Cumm%: 61.81 Cumm%: 60.48 SC: Kota SC: Kota
SC: Kota SC: Jaipur Central SC: Udaipur SC: Kota SC: Kota Cumm%: 74.88 Cumm%: 62.85
SAFAL- MR (Target AIEEE 2012) Cum.% up to PT-3
Cumm%: 88.75 Cumm%: 86.78 Cumm%: 86.16 Cumm%: 84.52 Cumm%: 82.66
Rank - 1 Rank - 2 Rank - 3 Rank - 4 Rank - 5
Rank Name Batch SC Cum.% Rank Name Batch SC Cum.%
6 Rishabh Solanki KTJRRJ1 Kota 82.1 50 Madhur Vyas KTJRRJ1 Kota 73.12
7 Vikas Garg KTJRR1 Kota 81.28 51 Nishant Yadav KTJRR1 Kota 73.08
8 Jagriti Singh KTJRRJ1 Kota 80.97 52 Vaibhav Gupta KTJRG1 Jaipur Central 73.06
9 Abhishek Kumar KTJRRJ1 Kota 80.87 53 Sarthak Agarwal KTJRR1 Kota 73.04
10 Mradul Dhakar KTJRRJ1 Kota 80.71 54 Shilpa Roy Lucknow - Alambagh 72.96
11 Shantanu Sharma KTJRRJ1 Kota 80.58 54 Ayushi Singhal Lucknow - Hazaratganj 72.96
12 Hemant Singh Jadon KTJRR1 Kota 78.62 56 Ganpat Choudhary KTJRG1 Kota 72.71 BHUPEN BHATNAGAR HIMANSHU VERMA ANUJ KUMAR BHUWAN BHASKAR NIRAJ KUMAR
13 Parth Nikhil Pandya KTJRRJ1 Kota 78.26 56 Shweta Shrivastava KTJRR1 Kota 72.71 Batch: KTMRR1, SC: Kota Batch: KTMRR1, SC: Kota Batch: KTMRR1, SC: Kota Batch: KTMRR1, SC: Kota Batch: KTMRR1, SC: Kota
14 Sagar Raj KTJRRJ1 Kota 78.24 58 Priyank Bisht KTJRRJ1 Kota 72.68 Cumm%: 79.33 Cumm%: 78.13 Cumm%: 73.63 Cumm%: 73.58 Cumm%: 73.45
15 Shubham Sharma KTJRRJ1 Kota 77.91 59 Vishal Choudhary DLEJR-2 Delhi-East 72.54
16 Surbhi Kumari NA Kota 77.36 59 Shyam Sundar ChoudharyKTJRRHR1Kota 72.54 Rank Name Batch SC Cum.% Rank Name Batch SC Cum.%
17 Rahul Gupta KTJRGH1 Kota 77.24 61 Arihant Agrawal KTJRRHR1 Kota 72.36 6 Ankit Kumar Tebrawal KTMRR1 Kota 73.32 8 Sulochana Saraswat KTMRRH1 Kota 71.83
18 Prakash Gupta KTJRRJ1 Kota 77.09 62 Mayur Khandelwal KTJRRHR1 Kota 72.24 7 Niraj Kumar KTMRR1 Kota 72.69 9 Mrityunjay Kumar KTMRR1 Kota 71
19 Vishal Goyal Lucknow - Hazaratganj 77.05 63 Siddharth Bakshi MBNEJRRN1Mumbai - Nerul 72.22 10 Manoj Kumar Mittal KTMRRH1 Kota 70.38
20 Kashyap Tibrewal KTJRRJ1 Kota 76.99 64 Vikas Jain KTJRR1 Kota 72.15
21 Sanjay Beniwal KTJRRJ1 Kota 76.73 65 Kumar Aniket KTJRRJ1 Kota 71.94 Resonance Yearlong Classroom Contact Programmes (YCCPs) - AIEEE Division
22 Abhishek Jha KTJRRJ1 Kota 76.51 65 Akshay Choudhary DLNWJR Delhi-North-West71.94
23 Binod Kumar KTJRR1 Kota 76.32 67 Shyam Sunder KTJRRHR1 Kota 71.93 ABHINAV-EA (Target AIEEE 2013) AJAY- ER (Target AIEEE 2012) Cum.% up to PT-3
24 Deepesh Yadav KTJRGH1 Kota 76.3 68 Mohit Bansal Jaipur Central 71.85 Cum.% up to PT-7 Rank - 1 Rank - 2 Rank - 3 Rank - 4 Rank - 5
25 Abhishek Goel KTJRRJ1 Kota 76.27 68 Neelam KTJRGH1 Kota 71.85 Rank - 1
26 Ankush Kothari KTJRRT5 Udaipur 76.18 70 Nalin Dhameliya Jaipur Central 71.67
27 Vaibhav Gupta DLSJR-2 Delhi - South 75.47 71 Nikhil Kumar Jaipur Central 71.52
28 Ankit Gupta KTJRGH1 Kota 75.41 72 Ankita R. Naik KTJRRJ1 Kota 71.51
29 Shivanshu Kumar Lucknow - Hazaratganj 75.38 73 Khatik Amit Usman KTJRRJ1 Kota 71.5
30 Chirag Agrawal KTJRRJ1 Kota 75.36 74 Amit Kumar KTJRR1 Kota 71.46
31 Gaurav Shukla KTJRRJ1 Kota 75.28 75 Sagar Kumar Garg KTJRR1 Kota 71.38
32 Aditya Kumar KTJRG1 Kota 75.19 76 Rajkumar Thakur KTJRR1 Kota 71.36 ANURAG DWIVEDI PRABHAT KUMAR ANSHU JAIN SAURABH PUROHIT AMIT KR AMAN
33 Amit Kumar KTJRGH1 Kota 75.15 77 Kushal Srivastava Lucknow - Alambagh 71.35 SURBHI GOYAL Batch: KTERR1 Batch: KTERR1 Batch: KTERRH1 Batch: KTERR1 Batch: KTERR1
34 Vikram Singh Pariyar KTJRRJ1 Kota 74.67 78 Naman Singla KTJRRU7 Kota 71.28 Batch: KTEAA1, SC: Kota SC: Kota SC: Kota SC: Kota SC: Kota SC: Kota
35 Harsh Vijay Jaipur Central 74.46 79 Yatin Gupta KTJRGH1 Kota 71.18 Cumm%: 71.54 Cumm%: 86.2 Cumm%: 82.5 Cumm%: 81.85 Cumm%: 80.42 Cumm%: 79.03
36 Anshu Sharma KTJRRJ1 Kota 74.41 80 Ankur Kislay KTJRRJ1 Kota 71.14
37 Ankit Mehta MBADJRRA1Mumbai - Andheri74.33 81 G. Ashrith Reddy MBNEJRRN1Mumbai - Nerul 71.02 AADHAAR-EB (Target AIEEE 2013)
38 Shashank Kamdar KTJRR1 Kota 74.3 82 Ankit Deora KTJRR1 Kota 70.87 Cum.% up to CT-1 Rank Name Batch SC Cum.%
39 Nikhil Kumar KTJRRU5 Kota 74.02 83 Mohit Kedia Kota 70.83 Rank - 1 Rank - 2 6 Vivek Gupta KTERR1 Kota 77.64
40 Gopal Shrama Jaipur Central 73.92 84 Dheeraj Khatri KTJRRHR1 Kota 70.65 7 Sanjib Karmakar KTERR1 Kota 76.57
SURABHI PORWAL AYUSH TABHANE
41 Abhishek Arora KTJRRJ1 Kota 73.88 85 V Govind MBADJRRA1Mumbai - Andheri70.57 8 Mukesh Kumar Koori KTERRH1 Kota 73.89
Batch: NPEB4 Batch: NPEB1
42 Naman Singhal KTJRR1 Kota 73.84 86 Kuldeep Singh KTJRGH1 Kota 70.55 9 Rajesh Kumar Jat KTERRH1 Kota 73.47
SC: Nagpur SC: Nagpur
43 Sambit Padhi Lucknow - Alambagh 73.77 86 Divjesh Singh KTJRR1 Kota 70.55 10 Ajay Raj Saini KTERRH1 Kota 72.78
Cumm%: 73.44 Cumm%: 69.75
44 Sandeep Maharia KTJRRJ1 Kota 73.73 88 Aditi Gupta DJEJR-2 Delhi - East 70.44 11 Mamraj Choudhary KTERRH1 Kota 72.71
45 Mangi Lal KTJRRHR1 Kota 73.55 89 Shantanu Sharma KTJRRJ1 Kota 70.39 Rank - 3 Rank - 4
12 Babu Lal Yadav KTERRH2 Kota 72.69
46 Aravind SomasudaramMBADJRRA1Mumbai - Andheri73.23 90 Shruti Garg KTJRR2 Kota 70.32 SHRUTI BHIWAPURKAR SHUBHAM YERAWAR 13 Vineet Kumar Delhi - South 71.94
47 Devendra Gupta KTJRRHR1 Kota 73.21 91 Vipul Goyal KTJRRHR1 Kota 70.27 Batch: NPEB1 Batch: NPEB1 14 Ashok Pal Jadoun KTERR1 Kota 71.11
48 Pappu Lal Dhaker KTJRRJ1 Kota 73.2 92 Abhilash Sharma Jaipur Central 70.25 SC: Nagpur SC: Nagpur 15 Subhash KTERRH1 Kota 70.46
49 Chetan Prakash Gupta KTJRGH1 Kota 73.13 93 Vaibhav Agrawal KTJRRJ1 Kota 70.22 Cumm%: 66.94 Cumm%: 62.32

ANUNAAD - XXV (October-December 2011) 24 ANUNAAD - XXV (October-December 2011) 25


Performance Matters
Winners From
Resonance Pre-Foundation Career Care Programme (PCCP)
VIGYAAN (Class: X, Phase-I) The All Centre Rank (ACR) is based on PT-7
Rank - 1 Rank - 1 Rank - 3 Rank - 4 Rank - 5

VINOD (Class: VII, Phase-I) The All Centre Rank (ACR) is based on PT-7
Rank - 1 Rank - 2 Rank - 3 Rank - 4 Rank - 5

KUSHAL BABEL SARVESH CHANDOLIA SAHOO ABHISHEK RAJ SHAURYA ARYA ANUBHAV SHARMA
Batch: UXE-1, SC: Udaipur Batch: XL, SC: Kota Batch: BXV, SC: Bhopal Batch: XL, SC: Kota Batch: AJX, SC: Ajmer
Cumm%: 94.44 Cumm.%: 94.44 Cumm%: 90.83 Cumm%: 90.28 Cumm%: 88.89
Rank - 5 Rank - 5
CHAVI HARKAWAT TUSHAR GUPTA KUSHAGRA JAIN AAKASH MATHUR SAQIB MOHAMMED
Batch: UWE-1, SC: Udaipur Batch: WE-1, SC: Kota Batch: WE-1, SC: Kota Batch: RWE, SC: Kota Batch: WE-1, SC: Kota
Cumm.%: 92.78 Cumm%: 90.28 Cumm%: 88.06 Cumm%: 87.5 Cumm%: 86.39 Rank Name Batch SC Cum.
8 Sujith Pillai BXV Bhopal 88.61
Rank Name Batch SC Cum. Rank Name Batch SC Cum.
9 Sidharth Agrwal UXE-1 Udaipur 88.33
6 Garvil Singhal WE-1 Kota 86.11 9 Ridhi Jain UWE-1 Udaipur 79.17
10 Mohit Bajaj XL Kota 88.06
7 Aashima Mittal WE-1 Kota 83.06 10 Vijay Tadikamalla NCK Nagpur 78.89
8 Shubhank Saxena WE-1 Kota 82.22
HIMANI PARTH SHARMA
VIMAL (Class: VIII, Phase-I) The All Centre Rank (ACR) is based on PT-7 Batch: RXE-1, SC: Kota Batch: SXE-1, SC: Kota
Rank - 1 Rank - 1 Rank - 3 Rank - 4 Rank - 5 Cumm%: 88.89 Cumm%: 88.89

VIBHAV (Class: VII, Phase-II) The All Centre Rank (ACR) is based on PT-3
Rank - 1 Rank - 2 Rank - 3 Rank - 4 Rank - 5

KAVYA BHANDARI CHIRAKSH SADAYAT AASHISH SINGH PRAKHAR BAFNAJ AKSHAT MAHESHWARI
Batch: UZE-1, SC: Udaipur Batch: JDZ, SC: Jodhpur Batch: ZE-1, SC: Kota Batch: DZ, SC: Jodhpur Batch: RZE-1, SC: Kota
Cumm%: 95.83 Cumm.%: 95.83 Cumm%: 94.72 Cumm%: 93.33 Cumm%: 92.5
Rank Name Batch SC Cum. Rank Name Batch SC Cum. ARPIT GUPTA JAYANT KUMAR GOYAL DHANANJAY KAJLA SASHAKT TRIPATHI DEEPANSH KASHYAP
6 Naman Jain ZE-1 Kota 92.22 9 Kunal Varshney RZE-1 Kota 90.83 Batch: KE-1, SC: Kota Batch: KE-1, SC: Kota Batch: BK-1, SC: Bhopal Batch: KE-1, SC: Kota Batch: KE-1, SC: Kota
6 Divyanshu Mittal ZE-1 Kota 92.22 10 Akshat Mittal UZE-1 Udaipur 90.56 Cumm.%: 91.67 Cumm%: 88.89 Cumm%: 77.22 Cumm%: 74.44 Cumm%: 73.61
6 Kratika Bhagtani ZE-1 Kota 92.22 10 Sarthak Sharma ZE-1 Kota 90.56
Rank Name Batch SC Cum. Rank Name Batch SC Cum.
6 Abhinav Singh KE-1 Kota 70.56 9 Siddhartha Nayak KE-1 Udaipur 64.44
VINAY (Class: IX, Phase-I) The All Centre Rank (ACR) is based on PT-7
7 Aman Gupta KE-1 Kota 69.72 10 Suryansh Kumar KE-1 Bhopal 63.89
Rank - 1 Rank - 2 Rank - 2 Rank - 2 Rank - 5 Rank - 5 8 Mitali Shukla KE-1 Kota 69.17 10 Urlam Sri Siri Spandana KE-1 Kota 63.89

VINAMRA (Class: VIII, Phase-II) The All Centre Rank (ACR) is based on PT-3
Rank - 1 Rank - 2 Rank - 3 Rank - 4 Rank - 5

ANUPREKSHA JAIN CHARVI NAHAR IRFAN Mohd. KHILJI RAUNAK NANDY SAMPRATI AGRAWAL SHUBHAM YADAV
Batch: YL, SC: Kota Batch: UYE-1, SC: Udaipur Batch: RYE-1, SC: Kota Batch: YL, SC: Kota Batch: BYV, SC: BhopalBatch: YE-1, SC: Kota
Cumm%: 94.72 Cumm.%: 93.06 Cumm%: 93.06 Cumm%: 93.06 Cumm%: 91.94 Cumm%: 91.94
Rank Name Batch SC Cum. Rank Name Batch SC Cum. JUHI MITTAL CATHY THOMAS AVIRAL JETHALIA SIDDHANT KEHRI NIMIT VIJAY
7 Prameet Singhi UYE-1 Udaipur 91.67 9 Prachi Jain YL Kota 91.11 Batch: NE-1, SC: Kota Batch: SNE-1, SC: Kota Batch: NE-1, SC: Kota Batch: BN-1, SC: Bhopal Batch: NE-2, SC: Kota
7 Bhavya Choudhary YL Kota 91.67 10 Sahil Mantri UYE-1 Udaipur 90.83 Cumm%: 84.72 Cumm.%: 78.33 Cumm%: 76.39 Cumm%: 76.11 Cumm%: 75.28

ANUNAAD - XXV (October-December 2011) 26 ANUNAAD - XXV (October-December 2011) 27


Phenomenal Past
Rank Name Batch SC Cum. Rank Name Batch SC Cum.
6 Ayush Sharma NE-1 Kota 73.61 8 Rajdeep NE-1 Kota 72.78
Fascinating Future
7 Sudhanshu Iyer SNE-1 Kota 73.33 9 Anirudh Ranawat UZE-5 Udaipur 72.5
10 Shubham Singh BN-1 Bhopal 72.22

VINEET (Class: IX, Phase-II) The All Centre Rank (ACR) is based on PT-3
October - December January -March April -June July - September
Rank - 1 Rank - 2 Rank - 3 Rank - 4 Rank - 5 Rank - 5 2005 2006 2006 2006

se rejouit 6
JATIN ARORA KULDEEP MEENA KARTHIK KUMAR AKSHAY S. THAKUR TUSHAR PARKAR VANDIT SAWANSUKHA
annees de
Batch: ME-1
SC: Kota
Cumm%: 89.44
Batch: ME-1
SC: Kota
Batch: ME-1
SC: Kota
Cumm%: 86.39
Batch: BM-1
SC: Bhopal
Cumm%: 78.33
Batch: AME-1
SC: Ahmedabad
Cumm%: 74.72
Batch: UYE-5
SC: Udaipur
Cumm%: 74.72
succes October -December
2006
January - March
2007
April -June
2007
July - September
2007
Cumm.%: 89.17
Rank Name Batch SC Cum. Rank Name Batch SC Cum. A unique endeavor of Resonance,
7 Rahul V. Finavia ME-1 Kota 73.61 9 Mayur Mohapatra AME-1 Ahmedabad73.06 ANUNAAD- a quarterly magazine,
8 Aayushi Nigam BM-1 Bhopal 73.33 9 Deepshikha Meena BM-1 Bhopal 73.06 released on 23 oct. 2005 with an aim
9 Vaishali Khadelwal ME-1 Kota 73.06
to ignite the spark of trust, confidence
VIPIN (Class: X, Phase-II) The All Centre Rank (ACR) is based on PT-3 and faith in the students & to
enlighten them through tricky and
Rank - 1 Rank - 2 Rank - 3 Rank - 4 Rank - 5 witty questions. Till date ANUNAAD
October - December January - March April - June July - September
has had the cover stories of some 2007 2008 2008 2008
iconic figures like that of Steel giant,
L.N. Mittal, Mr. Stephen Hawking
(physicist),
Ms. Sunita Williams, Mr. Mohnish
Pabrai (most promising of a new
RAJAT CHATURVEDI RAHUL KUMAR SARANSH CHAUHAN ANMOL YADAV SHUBHAM NAGARIA generation of value investors who had
Batch: BL-1 Batch: LE-1 Batch: SLE-1 Batch: SLE-1 Batch: ALE-1 lunch with Warren Buffet), Mr. Ratan
SC: Bhopal SC: Kota SC: Kota SC: Kota SC: Ahmedabad
Tata, Mr. Arun Sarin (Senior Adviser at October - December January - March April - June July - September
Cumm%: 87.5 Cumm.%: 75.83 Cumm%: 75.56 Cumm%: 74.17 Cumm%: 73.33
the private equity firm KKR). Ms. 2008 2009 2009 2009
Rank Name Batch SC Cum. Rank Name Batch SC Cum. Indra Krishnamurthy Nooyi
6 Harshit Chandaliya UXE-5 Udaipur 73.06 8 Neelabh Suman UXE-5 Udaipur 70 (Chairperson and CEO of PepsiCo
7 Prakher Gandhi UXE-5 Udaipur 71.94 9 Ankit Joshi ALE-1 Ahmedabad 68.61
10 Jayant Kumar Nema SLE-1 Kota 66.39
Inc.), Mr. D. Subbarao (Governor RBI),
Mr. Ajit Jain (CEO Berkshire
Hathway), Ms. Chanda Kochhar
(M.D. ICICI Bank), Mr. Nandan
Nilekani (Co-founder of Infosys and
C h a i r m a n U I D A I ) , M r. A r u n October - December January - March April - June July - September
2009 2010 2010 2010
M a j u m d a r ( D i r e c t o r, E n e r g y
Conservation, USA), Mr. Mahesh
Gupta (Founder, Kent Water Purifier)
and Mr. Nitin Nohria (Dean, Harvard
Business School) & Kiran Majumdar
Shaw.

ANUNAAD - XXV (October-December 2011) 28 October - December January - March April - June July - September
2010 2011 2011 2011
Total Selections 1816 in IIT-JEE 2011
(Yearlong Classroom Contact Programmes: 1197, Distance Learning Programme: 619)

SITTING ROW: SHUBHAM MAHESHWARI [1326], MAYANK GOYAL [1078], WADDEPALLI DURGESH NISHIKANT [614 (OBC-0052)], RAJENDRA SINGH [638], VAIBHAV GUPTA [526], RAHUL JAIN [356], MOHIT SENAPATY [336], AMAN SUNDERKA [246], SHUBHAM JAIN [106], SHUJAAT ISHAQ [199], UTKARSH SANJAU PATANGE [150], VAIBHAV DEVMURARI [113], GAURAV AHUJA [95], ANMOL GARG [66], MOHIT GUPTA [11], DIVYAM BANSAL [31], MRIDUL GARG [23], SHUBHAM MEHTA
[LEFT TO RIGHT] [2], MR. AYUSH GOYAL [HOD-MATHS], MR. CHANDRA SHEKHAR SHARMA [HOD-PHYSICS], MR. LOKESH KHANDELWAL [PRESIDENT-BUSINESS STRATEGY], MR. R.K. VERMA [MANAGING DIRECTOR & CEO], MR. ASHEESH SHARMA [VICE PRESIDENT-BUSINESS STRATEGY], MR. SHISHIR MITTAL [VICE PRESIDENT-STUDENTS WELFARE: BOYS & HOD CHEMISTRY], MRS. VIJAY PRATIMA MITTAL [VICE PRESIDENT- STUDENTS WELFARE: GIRLS],
NISHEETH LAHOTI [5], ARCHIT GUPTA [14], MEHUL GOYAL [26], VIVEK AGARWAL [35], GAURAV POONIWALA [36], NEERAJ YADAV [57 (OBC-0004)], RAHUL SINGHAL [18], S.S. KAUSIK [87], RAKSHIT GAUTAM [115], SHIVANSHU AGARWAL [294], RAHUL BALANI [352], CHETAN KUMAR GARG [394], PRAVEEN DUBEY [503], DEV PRIYAM [517], AYUSH AGRAWAL [667], ARNAV PURI GOSWAMI [339], ROHIT MITTAL [1485], BHAVIK JAIN [968]

FIRST ROW: MOHIT JAIN [1135], TULSI JAIN [1276], ARCHIT NARANIWAL [1226], RAHUL KUMAR SHAH [1474], JAY PRAKASH [1418 (OBC-0144)], AJAY KUMAR MEENA [ST-0482], VIVEK BHARGAV [1370], ABHISHEK SINGHAL [1207], NIKHIL KUMAR [1282 (OBC-0130)], MAINAK SETHI [1250], SONIA RANI [SC-1182], NEETA MAHALA [5866 (OBC-0828)], SHIVANI SINGHAL [5922], DIMPLE BANSAL [1801], JAYA GUPTA [2436], KSHITA JAIN [1405], ANU AGARWAL [3599], NIKITA GUPTA
[LEFT TO RIGHT] [3090], SONAL MAHESHWARI [2148], TAPASYA TIBREWAL [1619], NIRUPAMA PATEL [2390 (OBC-0275)], SHIVANJALI YADAV [8409 (OBC-1317)], ANKITA GAGRANI [3377], JAGRITI SAHU [3524 (OBC-0446)], PRANEETA NEHRA [5488 (OBC-0758)], SAPNA MEENA [ST-0305], SONAKSHI VERMA [SC-1642], MEENAL GHATIYA [2517], ARCHITA BHANDARI [931], NAYANSHI JAIN [6584], MANISHA VARSHNEY [1003], POJA NEGI [ST-0338], ANANYA [680], MOULIK GUPTA
[2177], NAVIN LALWANI [1108 (OBC-0114)], AKSHAT JAIN [692], ARCHIT JAIN [1017], DESAI ANIKET SHANTARAM [1115], SACHIN KUMAR YADAV [689 (OBC-0062)], AYUSH JAIN [737], GOYAL DHAWAL VISHNUKUMAR [915], CHETAN PALIWAL [911], RAVI KUMAR ROSHAN [639 (OBC-0056)], ASHWINI JAIN [932], VISHWAS JAIN [1657], SAURABH GUPTA [1342]

SECOND ROW: SAMBHAV KUMAR [3887], PURSHOTTAM SHARMA [2517], KAPIL REGAR [SC-0504], TANAY BOBDE [2026], DHEERAJ ARORA [3920], VIPUL GUPTA [2709], ANMOL [4491], PARNEET SINGH DEOL [2510], GAURAN JAIN [1988], KUMAR PUSHPAM [2720], TEJAS JAYANTILAL LUNAWAT [4392], PRADHUMN SINGH BITHU [3259 (OBC-0406)], PRAMOD KUMAR MEENA [2251 (ST-0006)], GAURAV KUMAR GUPTA [2486], AMIT KULHARI [8261 (OBC-1289)], KAPIL CHOUDHARY
[LEFT TO RIGHT] [2762], RAJAN MANTRI [2287], SWROOP SINGH DEVAL [2427 (OBC-0282)], PRINCE KUMAR KESHARWANI [2894], NITIN KUMAR [OBC-1905], RISHIKESH MISHRA [2882], ABHISHEK JAIN [2840], ANKIT KUMAR GOENKA [2173], NIDHISH JAIN [2145], KARMVEER SINGH [1964], HIMANSHU BAREJA [1576], MICKY KUMAR [2724], RISHU GARG [2275], DHEERAJ GOYAL [2083], ADITYA POKRA [1780], TEJVEER YADAV [1588 (OBC-0170)], ARPIT JAIN [1821], DEVARSHI
WAGHELA [2575], KHEMCHAND GUPTA [2999], SUSHIL KUMAR SINGH [2895], PRIYANK GOYAL [1874], SIVA KUMAR SINDRI [1691], NARINGREKAR TUSHAR MAHENDRA [1898 (OBC-0222)], MOHIT HEMRAJANI [2707], YEWLE DURVANK PURUSHOTTAM [3052 (OBC-0382)], DEEPANSHU KUMAWAT [1826 (OBC-0210)], MAHALE VINEET SUBHASH [2646], KUNAL DAHIYA [1997], ROHIT BANSAL [3142], ANIRUDHA TOLAMABIA [3021 (OBC-0381)], KRIPLANI PAVAN
RAJKUMAR [4854], SIDDHARTH SOGANI [3026]

THIRD ROW: RAVI SHANKAR SHARMA [3228], ANURAG DHARMAWAT [4114], KESHAV KUMAR MEENA [4279 (ST-0011)], YASH SINGH CHAUHAN [4299], ROHIT GUPTA [4332], SURYAPRATAP PRADIP BABAR [4967 (OBC-0666)], ASHUTOSH GOYAL [4839], ASHISH SAVITA [3853 (OBC-0497)], SHIVAM GUPTA [4850], UJJWAL KANT NIRAJ [4216], UMESH PRAJAPAT [4394 (OBC-0589)], SAGAR SINGH [4268], NAGARJUN NARAYAN [4102], VIKAS VERMA [4172 (OBC-0540)], ABHISHEK
[LEFT TO RIGHT] BENIWAL [4847], SUBOD GOYAL [4191], KETAN JAIN [2728], AAKASH CHAKRAVORTY [5710], SUMIT PRATAP SINGH [3100], PRADYUN MEHRA [3015 (SC-0031)], GAURAV SINGH [3273 (OBC-0410)], PRADEEP KUMAR [ST-0350], VIKAS VERMA [SC-0384], GIRIDHAR VIDYADHAR KULKARNI [2690], SIDDHARATH JAIN [3691], RAHUL PARASAR [3690], SAURABH SANJAY BHALERAO [2685 (OBC-0330)], VINAY VILAS BAGUL [2648 (OBC-0325)], ANADA RAM [2563 (OBC-
0306)], ANKUR BANSAL [2712], MUKUL GOYAL [2101], NAMIT JAIN [3099], ADITYA MITTAL [2804], AVDESH DIXIT [1122], SHUBHAM BANSAL [3649], ARPIT MAHESHWARI [3345], SOURABH AGGARWAL [3869], NISHANT TIWARI [3876], PRASHANT GUPTA [3749], SHIVAM GUPTA [1294], KUNAL KUMAR SOLANKI [4702], VIKAS KUMAR JOSHI [3906], SANDEEP PATIDAR [2449 (OBC-0287)], VIPIN AGRAWAL [3129], ANKUR KISLAY [4504], MOHAMMAD RASHEED
SANWARA [5480], SHAHID ARSHAD GAURI [6139 (OBC-0873)], PIYUSH DIXIT [4755]

FORTH ROW: GAURAV SHARMA [6597], ROUSHAN KUMAR CHANDAN [9552 (OBC-1527)], HIMANSHU KUMAR [6763 (OBC-0993)], SATYA PRAKASH YADAV [3567 (OBC-0450)], DHEERAJ KHATRI [5190], NITESH KUMAR RANKA [6059], NIKHIL JAIN [6398], PUSHPRAJ YADEV [5524 (OBC-0764)], RISHABH SHARMA [5424], ASHVINI KUMAR JINDAL [6754], PANKAJ GOYAL [5576], HARDIK AGARWAL [PD-113], SANDEEP KUMAR VERMA [6146 (OBC-0875)], AKSHAY DOSHI [5944], DIVYESH
[LEFT TO RIGHT] SANJAY KOTHARI [5203], RAJAT JAROLI [5539], MAYUR AGRAWAL [6030], NEETU MEENA [ST-0136], PRIYANKA YADAV [4075 (OBC-0528)], LALITA MEENA [ST-0524], REKHA MEENA [ST-0363], MEGHA AGRAWAL [5309], ASTHA AGARWAL [90], MALTI GUPTA [974], ALKA SINGH [SC-1056], ANITA BAI MEENA [ST-0540], ROHIT YADAV [5197 (OBC-0708)], SAURAV SINGH [5464], ASGAR AHMAD [8436], BANADA RAJAT RAMESHWAR [1589 (ST-0004)], ABHIMANYU
[4396], MOHIT RATHI [6169], SHUBHAM AGARWAL [5178], DUSHYANT SARASWAT [5047], NISHANT SHARMA [5428], ASHWINI KUMAR JANU [5848 (OBC-0825)], SUDHEER [5030 (OBC-0682)], AJAY KUMAR JAJOO [6803], ARPIT MALAV [7326 (OBC-1102)], ADITYA SINGH [6197 (OBC-0887)], DAKSH VERMA [6459 (OBC-0930)], MAYANK RAWAL [7612], DEEPAK KUMAWAT [6609 (OBC-0962)], SUDHATAM JAIN [6373], ABHIDEEP SAHU [6245 (OBC-0896)], PALASH
GUPTA [6977], NISHAN SINGH [6808], PIYUSH JAIN [6093], WASIM AKRAM SHEIKH [7709 (OBC-1178)], PINTU KUMAR MAHATO [PD-102]

FIFTH ROW: ADITYA SHARMA [8842], NAMAN DADHICH [8767], RAJEEV KHOJ [9545], LOKESH MEENA [ST-0256], SANJAY MANDA [OBC-1759], HEMRAJ KUMAWAT [8760 (OBC-1379)], DHARMENDRA KHICHAR [7592 (OBC-1153)], RAUNAK AGARWAL [9268], VASU PERIWAL [7571], KAPIL KUMAR [SC-1095], SANKIT MAROO [8080], SUNNY BHARGAVA [7229], AVINASH KUMAR GOYAL [7593], SUNIL KUMAR MATWA [6535 (OBC-0946)], MANDEEP [6724], VIKAS GARG [7739],
[LEFT TO RIGHT] ASHISH BALOTIYA [SC-1433], ARUN BHATI [6157 (OBC 0878)], AKASH MUTHA [6958], GAURAV KUMAR SINGHAL [8478], NITESH DAGUR [5704], DEEPAK AGRAWAL [6099], BABU LAL KUMAWAT [8085 (OBC-1254)], AJIT SINGH KASANA [9293 (OBC-1467)], VIJAY KUMAR MEENA [ST-0378], NARENDRA NAGAR [8112 (OBC-1261)], MADHAV KUMAR [PC], HARSHIT JAIN [9206], GOPAL SINGH [8433 (OBC-1319)], ASHISH KUMAR DOHARE [SC-1532], TUSHAR BEHALPADE
[8841 (OBC-1401)], DINESH KUMAR YADAV [ST-0065], HANUMAN PRASAD MEENA [ST-0047], UTKARSH VERMA [SC-0787], AMAN LOHIYA [9248], RAJENDRA PRASAD SINGARIA [SC-0895], HITESH KUMAR [SC-1943], ASHOK KUMAR [SC-0272], SURAJ KUMAR JAISWAL [8568 (OBC-1344)], ATUL KUMAR [ST-0288], RINKU MEENA [8017 (ST-0027)], AJAY RAI [8761], ARPIT MODANI [9289], SHUBHAM GOYAL [8127], KAPIL DEV SINGHAL [9210], MAHAMMAD UBAID [8942
(OBC-1416)], NIKHLESH RANJAN [PC], DEEPAK LOHAR [8915 (OBC-1411)], AMIT KUMAR [9128 (OBC-1443)]

SIXTH ROW: AKASH MEHRA [SC-0912], AKASH KUMAR NONIYA [SC-0221], VED PRAKASH MEENA [ST-0304], SATYENDRA PAL SINGH MEENA [ST-0321], PANKAJ KUMAR MEENA [ST-0372], MANOJ KUMAR MEENA [ST-0613], PRADEEP KUMAR [ST-0375], NITESH KUMAR MEENA [ST-0235], BANTU LAL MEENA [ST-0376], MOHIT PANWAR [SC-0139], DEEN DAYAL MEENA [ST-0366], PANKAJ KUMAR MEENA [ST-0591], MAHENDRA SINGH MEENA [ST-0431], RAY SINGH MEENA [PC],
[LEFT TO RIGHT] VIKASH KUMAR MEENA [ST-0121], RAVI RAJ MEENA [ST-0446], BALRAM MEENA [ST-0331], KRISHAN KUMAR MEENA [ST-0384], MANISH KUMAR MEENA [7161 (ST-0022)], BHEEM SINGH MEENA [ST-0278], NAMONARAYAN MEENA [ST-0160], HANSRAJ MEENA [ST-0118], PRATHVI RAJ MEENA [ST-0164], KULDEEP KUMAR MEENA [ST-0154], HITESH MEENA [ST-0170], HEMANT MEENA [ST-0421], PREM PRAKASH MEENA [ST-0507], PANKAJ KUMAR MEENA [ST-0190],
PRATEEK RAJ MEENA [ST-0087], JITENDRA MEENA [ST-0377], BRIJESH KUMAR MEENA [PC], VISHRAM MEENA [PC], RAJNEESH KUMAR MEENA [ST-0622], LOKESH MEENA [ST-0632], MANISH KUMAR MEENA [ST-0069], ABHISHEK KUMAR MEENA [ST-0495], JITENDRA KUMAR ALORIYA [SC-1539], RAVI MEENA [PC], RAJESH KUMAR MEENA [ST-0067], ASHOK KUMAR MEENA [ST-0345], NARAYAN LAL MEENA [ST-0189], ASHISH KUMAR VERMA [SC-0607], RAHUL
KUMAR [SC-0936], AMIT KUMAR MEENA [ST-0172], RAHUL DOHARE [SC-0753], KUNJI LAL MEENA [ST-0373], SANDIP VINODRAY RAVALIYA [SC 1692]

SEVENTH ROW: ANIL KUMAR [SC-0536], ABHISHEK DHAKAR [OBC-2224], JATIN RAI [OBC-1734], HITESH GARG [2649], DEEPAK KUMAR MEENA [PC], HARI SINGH MEENA [ST-0606], VIJAY KUMAR MEENA [ST-0505], VINAY PRATAP SINGH [SC-1169], JAYANT PATODIA [SC-1151], LOKENDRA KUMAR VERMA [SC-0583], AJAY MADHUKAR [SC-1126], CHETAN PRAKASH VERMA [SC-1424], HEMENDRA SINGH [SC-0702], BUDDHI PRAKASH MEENA [PC], RAI SINGH MEENA [ST-0131], MEETHA
[LEFT TO RIGHT] LAL MEENA [ST-0551], HENRAJ MEENA [ST 0155], ARJUN LAL BAIRWA [SC-0699], SAWAN KUMAR MEENA [ST-0358], MANISH KUMAR MEENA [ST-0477], MANOJ KUMAR VERMA [SC-0827], ABHISHEK UKEY [SC-0510], RAVISHANKER [SC-0227], MAYUR SURESHCHANDRA RAMTEKE [SC-1193], RANJEET KUMAR VERMA [SC-1910], DESH DEEPAK [SC-0840], HARSH KARMVEER [OBC-2113], CHIRAG NAMDEV [OBC-2131], DEWWRAT [OBC-2335], PREMCHAND GAUTAM
[SC-1842], VIVEK MEENA [ST-0626], MEENA MANOJ KUMAR [ST-0397], AJAY KUMAR SOYAL [SC-1831], HARIOM MEENA [ST-0218], YOGESH MEENA [ST-0498], PRAVEEN NAWAL [SC-0478], MONOJ JYOTI BASUMATARY [ST-0458], SURENDRS KUMARMEENA [PC], KALU RAM MEENA [ST-0466], MAST RAM MEENA [ST-0400], MANISH JYOTI BASUMATARY [PC], ANIL KUMAR MEENA [ST-0263], HANUMAN MEENA [PC], BRAJESH KUMAR MEENA [ST-0549]

EIGHTH ROW: UDAY SINGH [ST-0296], GAURAV MEENA [ST-0119], UMESH VANESA [SC-1033], SURAJ KUMAR MEENA [ST-0280], MANOJ KUMAR [SC-0818], MUKESH KUMAR [OBC-2529], SUNIL PANWAR [SC-1034], SUDHIR PANWAR [SC-0307], PAVAN KUMAR MEENA [ST-0195], HEMRAJ MENA [ST-0273], YASHODEEP PRABHU [OBC-2219], MANSIMRANJEET SINGH [5812 (OBC-0820)], HEMANT DANGI [SC-1325], RAVI CHOUDHARY [2888 (OBC-0357)], LOKESH KUMAR MEENA [ST-
[LEFT TO RIGHT] 0313], DEEPAK CHOUHAN [OBC-1882], JYOTI PRAKASH [OBC-1711], MOHAMMAD TAHIR [OBC-2140], RISHABH KUMAR DHAKAD [OBC-2216], LALIT KUMAR NAGAR [OBC-2409], RAHUL SURYAWANSHI [OBC-2320], GAJANAND YADAV [OBC-1588], JITENDRA KUMAR CHAUDHARY [OBC-1985], ARVIND KUMAR MEENA [PC], PRAMOD KUMAR [OBC-2127], RAJESH KUMAR MEENA [ST-0167], ABHAY KUMAR VIJAY [9090], DEEPAK KUMAR [OBC-1660], GOVIND DAN
CHARAN [OBC-2202], 1012647ROBIN SINGH GULSHAN [SC-1845], PAVAN VERMA [SC-1918], ASHOK KUMAR [SC-0149], ANKUR LUNIYA [SC-0282], DIPANSHU [SC-0528], SURENDAR JAIPAL [SC-0329], MADHAV VARI [PD-095], NARENDRA SINGH CHOUDHARY [SC-1410], PANKAJ SINGH [PD-059]
Design lessons from the Steve Jobs era Career Link
The name Steve Jobs is considered
synonymous with innovation,
functionality, aesthetic design and
perfection. The co-founder of Apple
consistently mesmerised the world with
technological marvels which have turned
popular culture,” as Steve Lohr wrote in
The New York Times.
Take a contradictory approach to
product designin other words, try to be
wildly imaginative and take grand risks
while always staying realistic. “The
KVPYKISHORE VAIGYANIK PROTSAHAN YOJNA
out so well, we couldn't have improved on lesson the world should take from Apple is The "Kishore Vaigyanik Protsahan Yojana" is an ongoing program started in 1999 conducted by the Department of Science
them even if we'd mapped our own lives that a company needs to become multi- and Technology, Government of India, to encourage students of Basic Sciences, Engineering and Medicine to take up
in advance. From the revolutionary dimensional. It needs to mix the core research careers in these areas.
Macintosh to the cult-status iPod, from the business with the disruptive innovation. It About a decade ago, the Department of Science and Technology (DST) felt absolutely convinced about the advantages of
scintillating iPhone to the awe-inspiring needs to combine the intellectual with the reviving a programme like NTSE, thus, the Kishore Vaigyanik Protsahan Yojana (KVPY) was initiated.
iPad, the technology czar has flummoxed artistic. It needs to maintain within it the
rivals and satiated consumers for around rational and the lunatic,” as Harvard The selection of students from those studying in +1, +2, any U.G. Program in Science / Medicine and also Engineering students having
three decades. Business Review blogger Horace Dediu aptitude for scientific research, are
The reaction to his death, with people wrote.
leaving candles and flowers outside Apple Design not only great products, but Funded By In Association With Zonal Centres Administered By
stores and the internet humming with also great marketing presentations of Department of Science and IIT-Bombay ICMR, IIT-Bombay, ICMR,
tributes from the world, is proof that Mr IIT-Bombay
these products. As Wall Street Journal Technology (DST), New Delhi, IISER - Kolkata New Delhi, IISER - Kolkata
Jobs had become something much more ICMR, New Delhi
reporter Yukari Iwatani Kane wrote, “Mr. Government of India. HBCSE, TIFR, Mumbai HBCSE, TIFR, Mumbai
significant than just a clever money-maker. Jobswidely known for his mercurial,
He stood out in three waysas a demanding management styledrove his
technologist, as a corporate leader and as company relentlessly to make products
Objectives of Examination Exam Pattern
somebody who was able to make people • The aim of the program is to identify and encourage talented The candidates have to go through the two stages to be selected
that consumers lusted for, unveiling them
love what had previously been students with aptitude for research. in KVPY.
in heavily rehearsed events that earned
impersonal, functional gadgets. Strangely, • It strives to assist the students to realize their potential and to Stage 1: Written aptitude test (multiple choices). The test will be
him a reputation as high-tech's greatest
it is this last quality that may have the ensure that the best scientific talent is developed for research conducted at different centers across the country.
showman.”
deepest effect on the way people live. The and growth in the country. Stage - 2: Interviews (Short listed Candidates from Aptitude Test
Ultimately, excellent design isn't
era of personal technology is in many ways • The mission is to attract exceptionally bright and highly will be called for the Interview which is the final Stage of the
about creating beautiful aesthetics; it's
just beginning. motivated students to research careers mainly in the areas of selection Process).
about creating memorable
basic sciences.
So what defines Jobs' design legacy. experiences. “The success of Apple, and Facts of KVPY 2010
Here are four design lessons that can be the design legacy of Steve Jobs, is [his] Eligibility No. of Candidates The cut off for both
learned from Steve Jobs' era, culled from laser-like focus on creating great consumer Stream
Basic Sciences Aappeared the streams is
some of the commentaries by journalists experiences,” as senior writer Jay Greene • Stream SA: Students enrolled in the XI Standard (Science 28,538 48%
and thinkers who cover Apple in the wrote on CNET. SA
Subjects) during the academic year 2011-2012 and have
context of design and its relationship to There have rarely been occasions in SB+2/SX 20,282 50%
secured a minimum of 80% (70% for SC/ST) marks in
both innovation and profit. What's history when one man has revolutionized aggregate in MATHEMATICS and SCIENCE subjects in the X
surprising, delightful, and practical (three an entire industry over and over again. Standard Board Examination.
Fellowships:
adjectives that can be used for Apple's Surely there have been myriad Monthly Annual
• Stream SX: Students enrolled in Class 12 (+2) of the Basic Sciences
products themselves) about these tips is innovations in science, technology, arts, Fellowship Contingency Grant
academic year 2011 - 2012 and aspiring to join
that they might not be obvious bits of literature, sports etc. but there hardly has undergraduate program in Basic Sciences (B.Sc/B.S./Int. SA and SP: (After X Class) Rs. 4000 Rs. 16000
product-design wisdom. They suggest that been a person who has consistently M.Sc) for the session 2012 - 2013 provided they secured a SX and SP: During B.Sc./
for Jobs, the beauty of creating Apple's reinvigorated a whole sector for decades. minimum of 80% (70% for SC/ST) marks in aggregate in B.S. and 1st to 3rd years Rs. 5000 Rs. 20000
products and services was about much He has been a standout performer in the MATHEMATICS and SCIENCE subjects in the X Standard of Integrated M.Sc.
more than engineering breakthrough times of cut-throat competition due to his Board Examination. SB and SP: During M.Sc./
software or designing sleek hardware. vision, drive and immaculate business • Stream SB: Students enrolled in 1st year B. Sc./B.S./Int. M.Sc. M.S and 4th and 5th years Rs. 7000 Rs. 28000
The formula for successful design? sense; and in doing so has carved a niche during the academic year 20112012 and have secured a of Integrated M.Sc.
Don't just focus on developing new for himself, endeared to billions in this minimum of 60% (50% for SC/ST) marks in aggregate in
tech; also pay attention to world and passed away with a permanent KVPY hopes that the award of the Fellowship to student is an
SCIENCE subjects in the XII Standard Board Examination.
entertainment trends. “[Steve Jobs'] heart-ache and left a permanent ding in encouragement, and opportunities that they derive from it will
design decisions…were shaped by his the universe. expose them to research in the subject of their liking and induce
understanding of both technology and them to choose a research career in science. This would really be
a fulfillment of the main objective of the KVPY program.

ANUNAAD - XXV (October-December 2011) 32 ANUNAAD - XXV (October-December 2011) 33


In this section Entrance to MBA & M.Des is given strong grip on fundamentals is essential earthquake engineering. A system of
through JMET and CEED respectively. IIT for the future engineers. Science-based material processing using focused beam
Kanpur, indeed, has a vibrant PhD courses are included in the curricula (FIB) has been established.
About IIT-Kanpur program in all the above academic areas as during the first two years of training so The Institute is expanding and various
well as in Humanities and Social Sciences. that the engineers follow a correct track in infrastructural developments have taken
Reso Alumni at IIT-K
The Department of Physics offers a M.Sc.- their innovative endeavor. Theodore von place in the recent past. The Outreach '69
Prominent IIT-K Alumni Ph.D. dual degree program, which allows Karmann once stated: The Scientists and '80 has come up which houses offices
Alumnus of IIT-K at Resonance their M.Sc. students to continue for a Ph.D. explore what is and engineers create what of Alumni Association and Students'
Featuring Mr. Lokesh Khandelwal The M.Tech and Ph.D students receive has never been. In IIT Kanpur, Science Placement activities. It also has a centrally
(LK Sir) research/ teaching assistantships. Electives have been introduced in order to air-conditioned multipurpose auditorium
enhance the science content in with the seating capacity of 218. The
The institute has a vibrant university engineering. The following Science Institute is setting up a centre of
atmosphere with a combination of strong Electives are worth mentioning: Quantum Environmental Sciences and Engineering
academic and research activities. Both Physics, Modern Theories of Material utilizing the MPLADS funds given by Shri
compliment each other and the students at Design, Non-Linear Systems, Order and Arun Shourie. The construction of a

IIT-KANPUR
all levels under the able guidance of the Chaos, Molecular Modeling, Bio- separate building is under advance stage,
faculty members maintain Inorganic Chemistry, Lasers in Chemistry which will house laboratories, seminar
this brilliant and Biology, Digital Image Processing, and discussion rooms for various

T
combination and disciplines of the
help the Institute to Understanding basic science and having a strong grip on fundamentals is Environmental
reach the zenith of essential for the future engineers. Theodore Von Karmann once stated: Sciences and
knowledge and The Scientists explore what is and engineers create what has never E n g i n e e r i n g.
he initiative to conceive a grand design for technical education in innovation. M a n y
To cite a few areas: developments
India and implement the concept true to its spirit, in the form of IITs is Finite Element have already taken
Fourier
Methods Using Domain place and many are lined up
one of the greatest hallmarks of visionary development in Decomposition, Flow Induced Vibrations,
Analysis, Numerical Linear Algebra,
for the future years. P. K. Kelkar Library is
Photons, Molecules and Chemical
independent India. Wind Tunnel Testing of Large Scale
Prototypes, Computational Chemistry,
Dynamics, Magnetic Resonance in
housed, with all modern amenities, in a
magnificent three-storied, separate
Chemistry and Biology, Bio-informatics
Nano-materials and Nano-technology, building covering an area of 5730sq. m.
and Computational Biology are to name a
Every year IIT Kanpur is enriched by the supported by U.S.A . Such close selected through JEE and usually they are Geometric Optimization of Large Organic The library has been rendering essential
few.
laurels brought by the faculty members interaction brought fresh air, new ideas the top students from various places in the Systems, Genomics and Bio-Informatics, support to the academic, research and
IIT Kanpur has developed a close
and the students in the form of path and novel thoughts into the academic country. There are programs for M. Sc. (2 Electronic Structure Calculations, developmental programmes of the
association with the Indian Railways. A
breaking innovations, research programmes and academic administration years) in Physics, Chemistry, Mathematics Aggregation and Etching, Molecular Institute.
new project namely SIMRAN has been
publications, projects, fellowships and IIT Kanpur substituted the traditional and Statistics, where the students with Dynamics, Thin Film Dynamics, Optical/
rolled out for tracking of trains and
industrial exposure. Established in 1960, annual system with semester system and B.Sc.(Hons) background are chosen EM Field Calculations, Computational
p ro v i d i n g t w o d i r e c t i o n a l d a t a
the Institute began functioning in the also introduced the letter grade system through an all-India examination known Fluid Dynamics and Heat Transfer,
connectivity between a stationary server
borrowed building of Harcourt Butler instead of marks for evaluating the as JAM. IIT-Kanpur has M.Tech Programs in Computer Aided Design and Rapid
and a moving train. This project has turned
Technological Institute in 1959 with 100 students for the first time in the country. all the Engineering Branches, mentioned Prototyping, Tomography, Robotics,
out to be a great success. The Research
students and a small faculty. The Institute IIT Kanpur also takes pride in introducing above. In addition, there are M. Tech. Multi-Body Dynamics, Geo-seismic
Summit called REACH Symposium was
now has its own sprawling residential the concepts of term papers, Programs in the interdisciplinary areas, Prospecting, Stress Analysis and
conducted at Shimla during March 2007
campus, about 2255 undergraduate and unannounced quizzes etc. such as, Nuclear Engineering, Biological Composite Materials, Vibration and
and is expected to be of great importance
1476 postgraduate students, 309 faculty Sciences and Bioengineering, Laser C o n t ro l , S e m i c o n d u c t o r P h y s i c s ,
of our Institute for bringing out new
members and more than 900 supporting At the Bachelor's level, IIT-Kanpur offers Technology, Environmental Engineering, Photonics, Neural Networks and Genetic
initiatives in academic and non-academic
staff. The combined record of its past and B . Te c h . P ro g r a m s i n A e ro s p a c e Materials Science, and Industrial and Algorithms, Earthquake Engineering,
environments. National Wind Tunnel
present faculty and students along with Engineering, Biological Sciences and Bio- Management Engineering. The M.Tech Impurities in Anti-Ferro Magnet, Raman
Facility (NWTF) was established as a
the alumni spread across the world is awe- Engineering, Chemical Engineering, Civil students are chosen through an all-India Scattering, Particle Physics, Spin
landmark research and development 3
inspiring. Engineering, Computer Science & examination, known as GATE. The institute Fluctuation in Quantum Magnets and so
activity at IIT Kanpur. Establishment of
Engineering, Electrical Engineering, has also adopted a dual degree (B.Tech- on. The most recent initiative of IIT Kanpur
Samtel Center for Display Technology is
IIT Kanpur had a massive collaboration Mechanical Engineering, Materials and M.Tech) Program in which, the students has been the Formation of a Strong
indeed a shining example of development
with USA through a specially created Metallurgical Engineering. It also offers admitted through JEE, are expected to Research Group in the areas of
of a research programme of high quality.
programme - KIAP (Kanpur Indo American integrated M.Sc. programs in Physics, complete the M.Tech Program in five Nanoscience and Nanotechnology.
The National Program on Earthquake
Programme). It is said to be the largest Chemistry, Mathematics and Statistics. years, at the end of which, the student is
Engineering Education (NICEE) is
ever academic assistance programme The students for these programs are awarded both B.Tech and M.Tech Degrees. Understanding basic science and having a
resource center for many aspects of

ANUNAAD - XXV (October-December 2011) 34 ANUNAAD - XXV (October-December 2011) 35


RESO ALUMNI PROMINENT
Aerospace Engineering
Name
Praveen Bhungar
Batch
G1
Civil Engineering
Name
Shashank Singh
Batch
GH1
Electrical Engineering
Name
Sanchit Goel
Batch
DP1
IIT-K ALUMNI
Prakhar Gupta PA1 Miss Anu Tripathi IP Hari Om Meena GH1 INDIA-GOVERNMENT AND ENTREPRENEURS (Abroad)
Varsha Yadav R2 Nivesh Goel LR1 Miss Pooja Singhal IP R&D Mr. Umang Gupta, Founder, Gupta
Virendra Mahawar RH6 Rahul Meena PA3 Mohit Agrawal JP1 Corporation
Dr. S. Sivaram, Director, NCL, Pune
Karanvir Singh DLPD Ramo Manjhi PA6 Abhiroop Bhatnagar JP1 Mr. Ravi Sethi, Presdient,Avaya Labs
Dr. H.S. Maiti, Director CGCRI, Kolkatta
Prakhar Gupta DLPD Dhanraj Meena RH2 Tanuj Sharma JP1 Mr. Abhay Bhuskan, several Start-ups
Dr. S.P. Mehrotra, Director, NML, Jamshedpur
Gurmeet Singh DLPD Jitendra Meena RH4 Suman Kumar PA1 INDUSTRY (India)
Dr. K. Vijayraghavan, Director, NBC, Bangalore
Prasoon Suchandra DLPD Kishalaya Kishor DLPD Gaurav Agarwal PA1
Dr. K. Balasubramanian, Director, NFTDC, Dr. S. K Kaura,CMD, Samtel Colour
Ravindra Rajaram Patil DLPD Shashi Prakash DLPD Abhishek Shreeram PA1
Hyderabad Mr. Som Mittal, CEO, Digital Global Soft
Ankur Pandey DLPD Anurag Meena PH1
Biological Sciences and Dr. Ashoke Sen, FRS, Mehta Institute, Mr. Suresh Pandey , MD, Bokaro Steel
Harshit DLPD Ram Lakhan Meena PH1
Allahabad Dr. B.N Singh, Former CEO, RINL and now
Bio Engineering Saurabh Dixit DLPD Harshit Khandelwal R1
Mr. Sudhir Vyas, IFS, Ambassador Jindal
Name Batch Eklavya Srivastava DLPD Nisha Choudhary R7
Mr. T. Ravi Mathur, Joint Secretary, MHRD Mr. Devasis Chowdhry, CMD, Midhani
Rohit Rajput G1 Anant Mundra DLPD Vinod Kumar Meena RH18
Mr. Rahul Asthana, formerly Best, now DAE Dr. Pawan Kumar Goenka, COD, Mahindra &
Siddharth Khera PA1 Abhinav Mishra DLPD Ravindra Kumar Sahu RJ1
Mahindra Ltd.
Pranjal Chandrakar DLPD Virendra Singh Bhawariya DLPD Shubham Khunteta RJ1
Puneet Mishra DLPD Deeksha Agarwal DLPD Utsav RJ1 ACADEMIA (India)
Naveen Kumar DLPD Rajkesh Meena RJ3 Professor Ashok Misra, Director, IIT Bombay INDUSTRY (Abroad)
Computer Science & Engineering Professor H.V. Sahasrabudhhe, Computer
Rajkesh Meena RJ3 Mr. Rakesh Gangawal, Former President, US
Chemical Engineering Name Batch Jitendra Narayan Verma DLPD Science, IIT Bombay
Airways
Name Batch Satish Kumar Meena FH1 Swati Tiwari DLPD Professor M. Agarwal, CSE, IIT Kanpur
Mr. Sanjay Mittal, CTO, Selectica, USA
Sumit Gupta DP1 Atique Firoz IP Pankaj Kumar Singh DLPD Professor P. Balaram, Chemistry, IISc,
Dr. Arindam Bose, President, Pfizer, USA
Dheerendra Singh G2 Niraj Kumar PA2 Arpana Yadav DLPD Bangalore
Dr. Rakesh Agarwal, President, Air Products,
Akshay Bansal JP1 Jay Prakash Meena PB4 Amish Goel DLPD Professor K. J Rao, Materials Science, IISc,
USA
Priya LR1 Swati R6 Harshal Chorrasiya DLPD Bangalore
Dr. Rajendra Singh, CEO, Telecom Ventures,
Aman Jain PA1 Abhishek Maheshwari RH1 Deepak Singh DLPD Professor H.R Krishnamurthy, Physics, IISc,
USA
Apurva Priyadarshi PB1 Banwari Lal Meena RH8 Ayush Jain DLPD Bangalore
Mr. Mukesh Pant, Reebok
Ashish Goyal RJ1 Pranjal Garg RJ1 Professor A. Chaudhari, Physics, IISc Bangalore
Kundan Kumar DLPD Materials And Metallurgical
Govind Khandelwal RJ1 Professor, A. Kumar, EE, IISc, Bangalore
Rahul Maji DLPD Engineering
Miss Shilpa Chhipa RJ2 Professor R. Sangal, IIT, Hyderabad More
Mohammad Shafiq DLPD Dhruv Lakhani DLPD Name Batch
Prabhu Goel, Chairman, Ipolicy Networks
Parth Shailesh Mehta DLPD Harshit Maheshwari DLPD Dholu Ram Yadav G1
Manav Garg DLPD Swati Varun G3
ACADEMIA (Abroad) Pradeep Sindhu, Vice Chairman, CTO &
Nishant Bhartia DLPD Founder, Juniper Networks
Abhishek Gupta DLPD Yash Choudhary JP1 Professor J. Malick, University of California,
Sindhukush DLPD Manoj Singh, CEO of the Asia/Pacific, Deloitte
Vibhav Agarwal DLPD Prashant Gupta PB1 Berkeley, USA
Mechanical Engineering Professor K. Pingli, Cornell University, USA Consulting
Chemistry Lokesh Kumar Yadav PH1
Professor Arvind, CSE, MIT, USA Anil K. Rajvanshi,1972 Director, Nimbkar
Name Batch Depanshu Mishra R1
Name Batch Professor J. Narayan, Materials Science, North Agricultural Research Institute and winner of
Ayush Anand G1 Devvrat Arya R3
Vivek Amarnani PA1 Carolina State University, USA Jamnalal Bajaj Award 2001 and FICCI award
Abhishek Sharma GH1 Vikas Soni RJ2
Meryl Likhita Lewis DLPD Professor M. Sur, MIT, USA 2002.
Pulkit Mittal IP Rahul Panwar DLPD
Nijit Sharma IP
Avijit Arya DLPD
Raghav Khandelwal DLPD Professor S.S Sahani, University of Florida, USA Debabrata Goswami, 1988 Scientist, Work
Professor A. Chakraborty, CHE, University of with Lasers; Wellcome Trust International
Sanjeev Kumar PA1 Economics Piyush Bajpai DLPD
Calfornia, Berkeley, USA Senior Research Fellow
Rohit Saini PB1 Name Batch Jainendra Batra DLPD
Professor J.A Sekhar, University of Cincinnati, Manindra Agarwal,1986, Clay Research
Aniket Sunil Mahapure PB3 Siddharth Sanghi JP1 Ankur Rajauria DLPD
USA Award, 2002, Godel Prize, 2006
Lokesh Jain R1 Jagdish. R. R1 Mathematics And Scientific Mriganka Sur, 1974, Professor of
Gaurav Kumar R1 Abhinav Sharma RH1 Computing Neuroscience, MIT
Anil Kumar RH2 Kanak Agrawal RJ1
Name Batch ENTREPRENEURS (India) Pradeep S Sindhu, 1974, Co-founder, Vice-
Himanshu Singh DLPD Kartik Pahwa RJ1
Sunil Kumar G3 Mr. N. R. Narayan Murthy, Chairman, Infosys President and CTO, Juniper Networks
Ajinkya Jain DLPD Physics Harsha Mulchandani JP1 Mr. Saurabh Shrivastav, Chairman, Xansa Corp.
Shaurya Shriyam DLPD
Name Batch Miss Jai Shri Rai PA1 Mr. Mohan Tambe, MD, Busmedia
Amrit Abhilash DLPD
Shashank Shekhar RJ2 Ankit Agrawal R1 Technologies
Akhilesh Garg DLPD
Shyak Bhattacharjee DLPD Priya Goel DLPD

ANUNAAD - XXV (October-December 2011) 36 ANUNAAD - XXV (October-December 2011) 37


What suggestions will you like to give to the students preparing for IIT-JEE at

Alumnus of Since I was preparing for IIT, my friends


used to ask me a lot of questions which I
Resonance to increase their productivity?
(i) There is no short cut to success, hence study with a plan, focusing on concepts.
(ii) Be regular in attending classes and give your best try at assignments before

IIT-Kanpur adds brilliance


coming to classes.
used to answer and that made me feel very
(iii) Believe in your teachers and study material, and always interact with your
happy. After graduating from IIT and
teachers if you require doing anything else in this regard.
working for some years at ABB and
(iv) Set a target while going for exams and try to do easy questions first.
to academic ambience at Resonance Infosys, I felt that I was becoming one
among many. At that time due to some (v) After exams, evaluate which questions you solved wrong or didn't attempt, and
Mr. Lokesh Khandelwal (LK Sir) family needs, I had to shift to Kota where I discuss with your teacher on how to improve on your performance.
B.Tech., (IIT-Kanpur, 1994) took to teach some students. Fortunately I
President (Strategy & Planning) got good response from my students and affects their studies, career & It was at IIT Kanpur when I opted for NSS
that boosted my confidence to take up eventually overall life? in I year. In this, we, a group of students
Resonance Eduventures Pvt. Ltd. teaching as a career. were required to go to a tribal district
I think youth should understand that time
A teacher is the second parent of a student. is scarce. One should try to set his priorities Jhabua in MP. We were supposed to teach
Distinctive features of your Teaching Methodology:
A student not only learns subject from his right and once it is done, think on other some basic calculations to villagers there
My teaching philosophy is actually pretty simple. I just play the dual role of a
teacher but also inculcate various other activities. For example, as a student I was a while staying with one of them. It was a
teacher as well as a student in class. Whenever I solve some question, I just think
traits from him, which may be his way of fan of cricket. During the preparations learning experience as I had to sleep on
that how a student from his knowledge base will think about solving it. I just focus
carrying himself - the way he talks, the way these full day cricket matches used to bare land, eat makka roti many a times
on connecting those links towards solution and thus help student to improve his
he thinks and much more. Therefore, a come on TV. I watched them only to have with red pepper and a pinch of salt.
thought process. Another important aspect is 'Listen to what question is saying
teacher not only has responsibility of guilt at the end of the day that I wasn't Further, I used to wait till evening when
and visualize the problem as far as possible.'
shaping a child's career but also his able to do my homework. Next time they came back from work. But the
personality to some extent. whenever match was there, I made up my enthusiasm that they had was just
Please tell us something about your
How has your teaching experience mind not to watch it and instead study. I awesome even after a full day of work in
early childhood and family was great to be a part of such an institute. Infrastructure facility at IIT-Kanpur.
been with the students preparing for just inquired the result & at the end of the jungles. It was a great experience for me.
background. Apart from regular courses, we use to have As I mentioned earlier, there are world
IIT-JEE at Resonance? day, I was very happy. I believe that Any book you read that transformed
I was born in Kota. My father and mother quizzes there which were open to all class labs, libraries, flying club having
students in campus. In one such The experience at Resonance is just students should develop a strong belief in your life. Please give us top learnings
both were born in villages near Kota. My couple of gliders, Olympic size swimming
Mathematics Quiz, we were pitched fabulous. You know that getting in IIT them and let me tell you, they and only from it or a brief review.
father worked in soil conservation pool, a small shopping area and a decent
against a team of immediate seniors requires excelling in all three subjects and they know where they are making a I didn't come across any such book as yet
department of Rajasthan Government. I Health center. In fact, I saw Electron
comprising JEE AIR-1 & AIR-3. There was it is here that I got great support from R K mistake and what is its solution. It is time but yes 3 idiots was a very inspiring and
did my X and XII from St. John's School, microscope there at our orientation to the
this last question which was related to Verma Sir, VPM Madam and many others. that you realize this latent power of yours transforming movie. It showed that life is
Kota. campus in the very first year. One of my
Ramanujan's number to which I gave an This, coupled with support from and pull up your socks for your career. not only about chasing grades or following
What inspired you for Indian Institutes friends did a number of hours in gliding
instant reply and we won the Quiz. It was and he was close to applying for next level management staff and great infrastructure What tips would you give to students the set rules but enjoying what you do and
of Technology (IITs)?
the talk of the campus for quite some time of being a pilot. brought out the best in me to give to my to exhale stress? doing what you enjoy.
My father, being an engineer, inspired me
which brought me and my team a lot of students. Enjoy your studies; try to give your best; Your interests & hobbies or anything
to be the same. Frankly, I didn't know What is the distinguishing factor of IIT-
pride. Tips on how to study Mathematics: study in one sitting or in small breaks but else that you love to indulge into
about IITs till I was in XI grade. IITs, being Kanpur which makes it different from
How is the academic and overall Maths is a subject which requires interest study with concentration. Try to do some besides teaching
most revered Technical institutes, naturally the other Engineering Institutes?
environment at IIT, Kanpur different and then you get addicted to it. It's like sort of physical activity an hour a day, be it I am a sports geek and play whichever field
became the coveted institutes I wanted to IIT Kanpur has the largest area among
from other IITs? meditation. When I solve Maths Problems jogging, swimming, working-out in gym, sports that I can lay hands upon. I also love
graduate through. engineering colleges in the country. Then,
at home, sometimes hours go by and I or a game of badminton. Sleep for at least to listen to old songs, especially Kishore da
What qualities one must possess to get As Kanpur campus is quite far from main I think it is its academic strength which sets
become oblivious of the world around me. 6-7 hours and most importantly be regular and Rafi.
into IITs? city and the city itself offers not too much it apart from the rest of IITs.
Be it loud music or a great cricket match, in your studies.
I believe that punctuality, determination of attraction, IIT Kanpur came to be What made you enter the field of Anything else that you feel worth
known for its academics. Frequent surprise nothing should come between you and Your motto of life/ideologies/values, sharing with all students.
and above all enjoying your subjects, are teaching? What is the responsibility of
quizzes and other activities, conducted by your problem-solving. The interest gets you live with
the keys to get in IITs. a teacher according to you? Ignore anything that comes between you
faculty members ensure that generated once you try to listen. Listen to Truthfulness and care are very basic virtues
Please share your experiences at and your goals because society doesn't
students do not lose focus on the problems, understand them, draw but forms core of me. I am also a green
What are the top five advices you would like to give respect you for your excuses but admire
IITKanpur and throw some light on diagrams wherever possible and collect
what they had come there for. person and feels that we should try our bit you for your achievements.
your best moments pertaining to to students for their lives to live. thoughts related to that problem. Attempt
Besides other facilities, the to save our environment.
studies there. Students should work hard in their career and enjoy all the problem and see if you enjoyed doing
academic environment is very Any life experience you wish to share
IIT Kanpur had a huge campus and vast the happiness and joy that life has to offer but should it, if not try some other method and even
well supported by institute's that tested your
infrastructure having best of labs, libraries know that apart from materialistic things, there are
Library, which is the size of a going from answers. Each of this will
some basic virtues that will go with you for long and will perseverance, amazing I was once in Ahmedabad where Shri Manmohan Singh,
and faculty members. It also has many football ground. develop a lot of skills in you which will
provide you respect not only from society but from spirit, resilience then Finance Minister, was the Chief Guest. The convener
extra curricular activities which includes help tackle any type of problems.
Tell us something about the yourself. These are honesty, truth, simplicity, towards hardships or there narrated a line which still is one of my favourites. He
almost all indoor and outdoor games. It What according to you are the core
campus and the compassion and respect to your parents, teachers your obstacles said “There is nothing which you and GOD together can't
seniors. problems of students/youth today that do.”

ANUNAAD - XXV (October-December 2011) 38 ANUNAAD - XXV (October-December 2011) 39


TALENT PHYSICS
Congratulations! HUNT FORCE
For Winning SILVER MEDAL
in Q.1 An exercise machine is designed to be a light weight cart on an inclined ramp on which a lady may lie and hold herself with two
parallel strings, which are attached to her cart through friction less pulleys. If the lady 600 N is able to pull and keep the strings
horizontal while the ramp is inclined at 30º, determine the face she is exerting by each hand. Find also reaction of the ramp on
the cart.

Jashan Singhal Yash Gupta Yash Nalwaya


Class : X Class : IX Class : X
Reso Roll No. : Reso Roll No. : Reso Roll No. :
(Held from 5th to 14th September 2011 at MODENA, ITALY) 840166 840107 1073267

Chemistry Olympiad Physics Olympiad Maths Olympiad


Q.1 When sulphur is boiled with Na2SO3, a Q.1 A current of 1.0 A is flowing in the 1. The number of positive fractions m/n
compound (A) is produced (A) with sides of an equilateral triangle of side such that 1/3 < m/n < 1 and having
excess of AgNO3 solution gives a 4.5×10–2 m. Find the magnetic field the property that the fraction
30º
compound (B) which is water soluble at the centroid of the triangle. remains the same by adding some
and produces a black coloured (Permeability constant µo= 4p× 10–7 positive integer to the numerator Q.2 A stone is dropped gently from the top of a tower during its last second of motion, it falls through 64% of the height. Find the
sulphide (C). Compound (A), (B) and V-s/A-m). and multiplying the denominator by height of tower.
(C) will be respectively : the same positive integer is :
(A*) Na2S2O3 , Ag2S2O3, Ag2S Q.2 A brass ring has a diameter db and an (A) 1(B*) 3 (C) 6 (D) Infinite
steel rod has a diameter ds(ds > db). If 2. A ray of light originating at the vertex
(B) Na2SO4, Ag2SO4, Ag2S Q.3 Two wire P & Q whose thermal coefficient of resistance aP = 4 × 105 /ºC and aQ = 8 × 10–5 /ºC, when connected in series their
the coefficients of linear expansion of A of a square ABCD passes through
(C) Na2S2O7, Ag2SO4, Ag2S equivalent thermal coefficient a’ is 6 × 10–5 /ºC. If these two wires are connected in parallel, what will be their equivalent
steel and brass are as and ab (ab> as) the vertex B after getting reflected by
(D) Na2SO5, Ag2SO5, Ag2SO4 thermal coefficient, a’.
respectively, by what temperature rise BC, CD and DA in that order. If q is
should both be heated so that the ring the angle of the initial position of the
Q.2 An organic acid (A) reacts with ray with AB then sinq equals : Q.4 Suppose molar heat capacity at constant volume of an ideal gas varies as :
fits over the rod ?
concentrated H2SO4 to give a neutral (A*) 2 / 13 (B) 3 / 13
oxide (B), acidic oxide (C) and a
(A*)
ds – db
(B)
ds  db (C) 3/5 (D) 4/5 Cv= a + b where P is pressure of gas and a & b are constants. Calculate equation of adiabatic process.
( )
diatomic oxide (D). When (D) reacts dbb – ds  s dbb – ds  s P
withchlorinegas,apoisonousgas (E)is
evolved. This gas with ammonia gives ds  db ds  db
an organic compound (F). The (C) dbb  ds  s (D) d   d 
b b s s
compounds (A) and (F) are :
(A*) (A)=H2C2O4 and (F) =NH2CONH2
(B) ( A ) = C H 3 C O O H a n d ( F ) =
NH2CONH2
(C) A) = CHCl3 and (F) = H2C2O4
(D) (A) = CCl4 and (F) = CH3CHO
Ashish Chawala
Reso PCCP Faculty Member, Maths

Richa Sharma
Reso PCCP Faculty Member, Physics

JKP Sir
Sangeeta Agarwal (For Solutions of Q. 1 to 4, please refer page 'Khul-Ja-Sim-Sim') Sr. Reso Faculty Member, Physics
Reso PCCP Faculty Member, Chemistry

ANUNAAD - XXV (October-December 2011) 40 ANUNAAD - XXV (October-December 2011) 41


TALENT CHEMISTRY
HUNT CATALYST
Methods to decide a reaction as SN1 or SN2. (with Examples) P Non-polar solvents. 3) Solvent: polar aprotic. Good for SN2.
P Not good for either SN1 or SN2. 4) Leaving group: NH2-, which is an extremely bad leaving
The given approach is useful to determine P SN1 reactions favored by weak P polarizable (able to stabilize the P Problem: nucleophiles not typically soluble in these solvents. group.
whether a chemical reaction occurs via a nucleophiles. transition state). Therefore, they are not really useful for substitution reactions. Decision:
SN1 or SN2 mechanism. This approach P What makes a nucleophile strong? P Weak bases that are common, good Everything points to a SN2 mechanism until the leaving group is
involves analysis of reaction conditions in Negative charge: leaving groups: STEP 5: Sum up the ideas. considered. In this case, the leaving group is very poor. Therefore,
details. You should analyze the following P A negatively charged species is a better P Cl-, Br-,I P Remember, nature of nucleophile and structure of substrate it is doubtful if this reaction will even work at all.
aspects of each reaction, in the order nucleophile than a similar, uncharged P Sulfonate, sulfate, phosphate are most important. Answer: No reaction.
introduced below. species. In particular, a base is a P Neutral molecules: P Nature of leaving group less important, since good leaving
stronger nucleophile than its conjugate P H2O groups are important for both mechanisms. CH3CN
c) CH CH CH CH CH I + NaOCH3
3 2 2 2 2
STEP 1: Check the Reactant acid. P Alcohols (R-OH) P Solvent not a major factor, but its identity (polar protic, polar
P Remember: the structure of the Electronegativity: P Amines (R3N) aprotic, etc) can be a deciding factor in your decision.
Analysis:
substrate is a ver y important P Nucleophilicity decreases from right to PPhosphines (R3P) 1) Substrate: primary. Good for SN2.
consideration in determining if a left on the periodic table, following the WORKED EXAMPLES:1
P THE FOLLOWING ARE NEVER GOOD 2) Nucleophile:OCH3-,Strong nucleophile, good for SN2.
reaction occurs by an SN1 or SN2 increase in electronegativity. LEAVING GROUPS: CH3 3) Solvent: polar aprotic. Good for SN2.
mechanism. Size and Polarizability: a)
P Hydroxide (OH-) CH3 OH 4) Leaving group: I-, a very goodleaving group.
P Remember the trend of substrate P Nucleophilicity increases down a group CH3 CH2 C CI + CH3 OH
P Alkoxide (RO-), like CH3CH2O- Decision:
reactivity for SN1& SN2: on the periodic table, following the -
P Amide (NH2 ) H Classic example of an SN2 reaction. Everything points to it.
SN1: 3º > 2º > 1º > methyl increase in size and polarizability.
Answer: SN2 reaction.
SN2: methyl > 1º > 2º > 3º P What makes a nucleophile weak? Analysis:
STEP 4: Check the nature of solvent
PIf your substrate is tertiary (3º ): SN1 P Neutral charge 1) Substrate: secondary. Can go either SN1 or SN2
P Solvent does play a role in determining d)
highly favored. It won't go by SN2. P Remember sterics! CH3OH
whether a reaction goes by a SN1 or SN2 2) Nucleophile: Weak nucleophile (it's neutral). + NaI
P If your substrate is secondary (2º ): Not P Bulky nucleophiles slow down reaction 3) Solvent: polar protic. Good for SN1. Br
mechanism. However, it is typically
the best substrate for SN1, but it still is rates. 4) Leaving group: Cl, which is a good leaving group.
not a major role… but it can “tip the
Analysis:
ok. Not the best for SN2 also, but it scales” in the favor of one
1) Substrate: primary. Good for SN2
could still go by this mechanism. Still STEP 3: Check the leaving groups Decision:
mechanism depending on the
P SN1 and SN2 reactions favor good 2) Nucleophile: I-, Strong nucleophile, good for SN2.
SN2 is favored as SN2 takes place at nature of the nucleophile and the Remember that substrate structure and the nucleophile play
leaving groups. Therefore, this aspect is 3) Solvent: polar protic. Good for SN1, not the best for SN2.
faster rate than SN1. In other words, substrate. major roles in this decision. The substrate is secondary, which
really not a major factor in deciding if a 4) Leaving group: Br-,a very goodleaving group.
it could go either way, so you must Polar protic solvents: can go either SN1 or SN2. Therefore, you must consider other
reaction follows a S N 1 or S N 2 Decision:
look at other factors before making P Have acidic hydrogens factors. The nucleophile is weak, which is good for SN1. The
mechanism. The data suggests SN2, except for the solvent. Methanol is a polar
your decision. P Very strong solvating power solvent is polar protic, which is also good for SN1. The leaving
P HOWEVER: the nature of the leaving protic solvent, which is good for a SN1 reaction. However, given
P If your substrate is primary (1º ): SN2 P Very good for SN1 reactions. group is good, which doesnot really matter here, but it does
group plays an important role in indicate that a substitution reaction (whichever mechanism) will the nature of the substrate and nucleophile (both good for SN2),
favored. It won't go by SN1. P Examples: water, alcohols
determining if a reaction will occur. the solvent won't be that important.
P If your substrate is methyl: SN2 highly Polar aprotic solvents:
undergo a substitution reaction at Answer: SN1 mechanism. Answer: SN2 reaction.
favored. It won't go by SN1. P Do not have acidic hydrogens
all!
P Good solvating power, but not as strong
P What is a good leaving group? acetone
STEP 2: Check the Nucleophile as polar protic solvents. b) CH3 CH2 CH2 CH2 NH2 + NaOH
P Electron withdrawing, to polarize the
P Is the nucleophile strong or weak? P Very good for SN2 reactions.
carbon atom on the substrate.
(alternate: good or bad). P Examples: acetone, acetonitrile, Analysis:
P stable (not a strong base) once it has
P SN2 reactions favored by strong dimethylformamide, crown ethers. 1) Substrate: primary. Good for SN2.
left. S. K. Sinha (SHK Sir)
nucleophiles. Aprotic solvents: 2) Nucleophile:OH-,Strong nucleophile, good for SN2. Sr. Reso Faculty Member, Chemistry

ANUNAAD - XXV (October-December 2011) 42 ANUNAAD - XXV (October-December 2011) 43


TALENT

1.
HUNT
MATHS
EXPONENT
If a and b (a < b) are the roots of the equation x2 + bx – c = 0, where b < 0 < c, then
Think To
Reso Q.
24
Win
The first correct answer will fetch a prize.
Submit your answers at J-2, Help Desk, Kota.
(A) 0 < a < b (B) a < 0 < b < |a| (C) a<b<0 (D) a < 0 < |a| < b

 Physics Chemistry
2. Let ABC be a triangle such that ÐACB = 6 and let a, b and c denote the lengths of the sides opposite to A, B and C respectively.
Q.1 is for foundation students and Q.1 is for foundation students and
The value(s) of x for which a = x2 + x + 1, b = x2 – 1 and c = 2x + 1 is (are)
Q.2 is open for all students Q.2 is open for all students
(A) – (2+ 3 ) (B) 1 + 3 (C) 2+ 3 (D) 4 3
Q.1 A body weighing 800 N is hung from a horizontal ring of Q.1 Organic compound “A” of molecular formula C9H9O3N
th diameter 6m by means of three cords, each 5m long. On reacts with organic compound “B” of formula C10H15N
3. The ratio of sums of n-terms of two arithmetic progressions is (3n - 13):(5n + 21). The ratio of 24 term of the two series is:
the ring, two of the cords are placed 90º apart and the followed by mild acidification and again washed with
(A) 59 : 141 (B) 7 : 17 (C) 1:2 (D) none of these point of attachment of the third cord bisects the mild base to form Labetatol. Write the structure of A and
 
remaining arc of the ring. Find the tension in each cord. B. (Ignore Stereochemistry).
k
4. The value of the sum is equal to  n=1
 2n+K Z
O OH
k=1
NH
H2N
(A) 5 (B) 4 (C) 3 (D) 2 Y
B
CH3
HO Labetatol
5. If 'l' is the length of median from the vertex A to the side BC of a DABC, then X
c A Q.2 Match the compounds given in column I with the pKa
(A) 4l2 = b2 + 4ac cos B (B) 4l2 = a2 + 4bc cos A (C) 4l2 = c2 + 4ab cos C (D) 4l2 = b2 + 2c2 – 2a2 data of their ammonium ions given in column II.
Column-I Column-II
pKa data of ammonium ions
6. The solution set of the inequality 4 (cos–1x)2 – 1 ³ 0 is T2 Compounds
of the compounds
T3 T1 A. CH3CH2NH2 P. 10.75
  , 2
(A) 1, cos 1 (B) 0, 1
(C) cos 2 , 1 (D) B. (CH3CH2)2NH Q. 11.27
2 3 3 3
800N
NH
7. Let A = {x | x2 + (m – 1) x – 2 (m + 1) = 0, x Î R} Q.2 An insulating long massless rod of length L pivoted at its C. R. 10.98
2 centre and balanced with a weight ‘W’ at a distance ‘a’
B = {x | (m – 1) x + mx + 1 = 0, x Î R}. N
from the left end, as shown in figure. Charges q and 2q D. S. 5.25
Number of values of m such that A È B has exactly 3 distinct elements, is are attached at the left and right ends of rod and at a
(A) 4 (B) 5 (C) 6 (D) 7 distance ‘b’ directly below each of these charges a N
positive charge Q is fixed. Find out the distance ‘a’ in E.
N
T. 1.3
terms of given parameters for equilibrium.
8. Through the centroid of an equilateral triangle a line parallel to the base is drawn. On this line, an arbitrary point P is taken inside
L
the triangle. Let h denote the distance of P from the base of the triangle. Let h1 and h2 be the distance of P from the other two S. K. Sinha (SHK Sir)
a
q 2q
Sr. Reso Faculty Member, Chemistry
sides of the triangle, then
OR
(A) h is then H.M. of h1, h2 (B) h is the G.M. of h1, h2 (C) h is the A.M. of h1, h2 (D) none of these
Q. 2 An aliphatic monoamine (vapour density of 36) on
b W
b treating with excess of CH3I for a long time, a quaternary
9. Let A = {9, 10, 11, 12, 13}and let f : A ® N be defined by f(x) = the highest prime factor of x. Then the number of distinct Q salt containing 55.5% of iodine is produced. Write the
Q
elements in the range of f is possible structure of amine.
(A) 6 (B) 5 (C) 4 (D) none of these JKP Sir Girijesh Dubey (GD Sir)
Sr. Reso Faculty Member, Physics Sr. Reso Faculty Member, Chemistry

Maths
Q.1 is for foundation students and Q.2 is open for all students
Q. 1 Let ABC be an acute triangle. Find the positions of the points M, N, P on the sides BC, CA, AB, respectively, such that the
perimeter of the triangle M N P is minimal.
Q. 2. Let A, B, C and D be four points in space, not in the same plane, prove that
Safdar Seraj (SS Sir)
(For Solutions of Q. 1 to 9, please refer page 'Khul-Ja-Sim-Sim') Safdar Seraj (SS Sir) AC . BD < AB . CD + AD . BC
HOD - Maths, Patna Centre Centre Head, HOD-Math, Patna Centre

ANUNAAD - XXV (October-December 2011) 44 ANUNAAD - XXV (October-December 2011) 45


Know Mania selecting the scholars: Participation in campus activities: real life academic competition, STaRT- Championís Camp at Kota, Rajasthan in
The 80 students from the IIMs and the 120 Active participation in a minimum of two 2012 is loaded with excellent & which expert mentors of Resonance shall
students from the IITs and BITS (Pilani) forums on campus biggest prizes & awards to the winning educate and guide them with a focus on
who meet the eligibility criteria are students. establishing foundation for future
evaluated on the basis of the information Learning: Scholar has to write a 350-word The unique feature of prizes in STaRT-2012 competitive exams like NTSE, IJSO, KVPY,
they provide in their application form. essay on "Being an Aditya Birla scholar: is that the finally declared National Olympiads, IIT-JEE and other Engineering
The students' overall achievements, experience sharing" toppers (1st Rank) of each class (total & Medical entrances according to
spanning academic and co-curricular seven students, one from each class) shall relevance of the class of students.
excellence, are weighted and assessed. win a Gold Medal & Eleven days visit to
Those selected for the next round are STaRT - 2012 USA including three days visit to Space According to the format of STaRT-2012,
asked to write an essay. Camp at Kennedy Space Centre, NASA, the schools can conduct first stage test of
Essays written by these students are USA and also four to five other tourist STaRT-2012 in their own premises and also
ABGS evaluated by a panel of judges and, based attractions in USA . The prize worth for 1st become entitled for awards of worth more
The Aditya Birla Group Scholarship on their performance, candidates are Rank will be more than Rs. 2 Lac to each than Rs. 12 Lacs (Rupees Twelve Lac) in
(ABGS) scheme, instituted in 1999, is a shortlisted for the final round. student. The National Runners-up (2nd various reward categories for Schools,
way of spawning leaders with a passion to Rank) of each class (total seven students, their Principals and Coordinators based on
excel. The Group fosters outstanding Those shortlisted are invited for an one from each class) shall win a Silver performance and participation of their
academic excellence and humane interview. Medal & Six Days visit to Singapore students.
leadership values, in partnership with The 10 best engineering and the 10 including SCIENCE CENTRE SINGAPORE
select institutions such as the IITs, BITS best management students become and also four to five other tourist A student, apart from his/her own school,
(Pilani), and the IIMs. The ABGS Aditya Birla Scholars. attractions in Singapore. The prize worth also has options to participate in first stage
programme is not just about academic The interviews are conducted by an for 2nd Rank will be more than Rs. 1 Lac to of STaRT-2012 by taking test either at
excellence; aspiring scholars are evaluated eminent panel of business luminaries and each student The National 3rd Rank of each Resonance test centre or Online.
on holistic parameters that take on board academicians in Mumbai. Invited Class (total seven students, one from each
their extra-curricular activities, leadership candidates are given an allowance to class) shall win a Bronze Medal & a high- The first stage test of STaRT-2012 shall
qualities, values, vision…their dreams. So cover expenses on travel and end Laptop of worth Rs. 50000 each. The be conducted on various dates in
far, 157 Aditya Birla Scholars have been accommodation. students from All India Rank from 04th to December, 2011 and the second stage
STaRT-2012 is a Scholarship cum Talent 10 in each class shall be rewarded with test shall be conducted on 22 January
incubated through this programme.
Reward Test, launched by Resonance. Desktop PCs worth Rs. 25000 each. The 2012. Both the tests will be OMR based,
The value of the scholarship for various The Aditya Birla Scholars are assessed
It is a two-stage nation-wide grand students from All India Rank 11to 500 comprising objective type questions from
institutions is as under: regularly on qualitative and quantitative
Academic Talent test for students of class shall also win prizes ranging from Personal Physics, Chemistry, Mathematics, Biology
IIM: Rs 1, 75,000 per annum parameters to judge their performance on
VI, VII, VIII, IX, X & XI, and XII of Science Music Players (PMPs), Sports Watches to & Mental Ability, mainly from NCERT
IIT / BITS (Pilani): Rs 65,000 per annum the academic and leadership front. This
Stream. The Stage-I will be qualifying branded Writing Pens. syllabus of the respective class. Class XI
To be eligible to apply for the a s s e s s m e n t f o rm s t h e b a s i s f o r
examination for the Stage-II and Stage II Through STaRT-2012, Resonance shall give and XII students of Mathematics and
scholarship, one must: determining the continuation of the
qualifiers will be awarded in a Felicitation away various attractive prizes & awards as Biology shall get different papers.
1. Gain admission to one of the following scholarship for the next academic session.
Function & Champions' Camp at Kota. mentioned above, having a total worth of
institutions: For the IIMs, the assessment is done once
Considering the prevailing level of more than Rs. 50 Lacs (Rupees Fifty Registration for STaRT-2012, has
IIMs: Ahmedabad, Bangalore, Kolkata in a one-year time frame.
competition in the country, a student lacs), to 3500 students in total (500 already opened on September 26,
and Lucknow For the IITs and for BITS (Pilani), the
cannot just plunge headlong into realizing students from each class). Resonance 2011 & would close on November 26,
IITs (B. Tech): Chennai, Delhi, Kanpur, assessment is done thrice in a three-year
his/her career dream without facing tough has also generously allocated scholarships 2011 whereas the complete information
Kharagpur and Mumbai time frame.
competition. In order to pierce forward at of maximum worth of Rs. 06 Crores and guidance for STaRT-2012 has been
BITS: Pilani every level, a student needs to test himself
Assessment criteria: (Rupees Six Crores) for the final top made available on STaRTís official website
2. Feature among the top 20 in terms of at appropriate bench-mark with right
Scholastic standards: The scholar must hundred students, from each class, if they www.resostart.in . The same is also
the entrance exam ranking at the time of strategy. STaRT offers a solid platform to
be in the top 25 per cent of students in e n ro l l i n t o y e a r l o n g c l a s s ro o m available on Resonance's website
admission. students so that they can judge themselves
his/her batch programmes of Resonance targeting CBSE, www.resonance.ac.in
How to apply on a national benchmark and also develop
Work done by the scholar during the NTSE, KVPY Olympiads, IIT-JEE, AIEEE,
Eligible students are required to fill the a spirit for forthcoming important formal
course of the programme: At least 60 AIMPT as relevant to their class and
application form and submit it at the given competitive & talent examinations in their
per cent of the assignments must have a aspirations. The winners will be invited at
address. Forms are available only through career.
rating of at least 7 on a 9-point scale Kota for the Felicitation Function to be
the institution. Along with the academic and self Manoj Sharma
Practical work-related experiences of held in February, 2012.The final top
development benefits for students like hundred students of all classes except class Vice-President
Selection procedure the scholar: Summer project must have a
National Benchmarking and exposure to XII shall also be invited for three days Operation & Business Development
The following process is followed in rating of at least 7 on a 9-point scale
Resonance Eduventures Pvt. Ltd.

ANUNAAD - XXV (October-December 2011) 46 ANUNAAD - XXV (October-December 2011) 47


RESO REAPER Archit Gupta needs to be determined about his goals. technology. My brother, Ankit Gupta works except an award for excellence in Sciences,
Father's Name: Shri M.K. Gupta Like a set of 60 problems that have to be for Techmahindra at Pune and we have a which I received from His Excellency, Dr. A.
Mother's Name: Dr. Shobha Gupta solved in 3 hours. Thereafter, go out and pet dog, a german shepherd, named Tuffy. P. J. Abdul Kalam. Before coming to Kota, I
Occupation of your Father: Service (Electrical Engineer) party hard! But set your goals first. It is It is great to have them all together. used to take part in lots of debates and
good to have a work plan, but it is useless cultural festivals too.
Reso Roll No.: 923350 Category: General
unless you implement it. Is there any IITian or engineers in your
IIT-JEE (AIR): 14 Course: VIJETA family? Did you get motivation from There are many who are going through
Home Town: Kota Study Centre: Kota Let's talk about your mentors at this? crucial preparations for IIT-JEE 2012.
Batch: IP Branch: Computer Science Resonance. Yes, there are engineers in my family, not Any advice for them?
Year: 2009-10, 2010-11 I had a liking for all the teachers who IITians though. My father did his Just work hard, enjoy your efforts. If you
taught me at Resonance. My initial batch graduation in Electrical Engineering and feel that you are incapable, I have a
First, accept the hearty congratulations millions of student running blindly
was BK8. I was there for a little over a m y b ro t h e r i n E l e c t ro n i c s a n d biography to narrate, the biography of
once again on your success at IIT-JEE towards these marvelous institutions. I
month. It was good to see my teachers Communication Engineering. Their Lance Armstrong.
2011. never really had a huge desire for
from the batch later on, when I had moved support and guidance was crucial for me to He was a cancer patient. Stage three, at the
Thank you so much. engineering but as it turned out, I built up
up. They had their expectations and I hope make my career choices. verge of death. His survival chances were
a liking for it.
I lived up to them. In class 12th I grew less than 20%. Cancer had spread into his
You achieved AIR (Gen.)-14 in IIT-JEE pretty fond of RKV Sir, ST Sir, SPR Sir and Any memorable event/incident at brain and lungs. He could've quit and died
2011. Did you ever imagine at any time What do you think is essentially VPM Ma'am. RKV Sir is simply amazing. like a million others do…
Resonance that you wish to share with
that you would be one among the TOP- required to get into IIT? Everything about him, right from the But he didn't. Today he holds the world
us
20 AIRs in one of the most grueling If a person is sure that he wants IIT, he can stories he narrates, the mistakes he points record of winning Tour de France for seven
We celebrated RKV Sir's birthday at our
engineering entrance exams in the get in without any troubles. Well, the city out and the way he scolds. A classroom, consecutive times. 2,290 mile cycling race
hostel - singing till one o' clock at night
world? where I studied and a lot of readers are bundled up with sheets, DPPs and all the that spans entire France.
with him, and the final words from Sir
Initially, the expectations were low. That studying today, is a cunning city. It is very fatal stuff becomes a lot more interesting Do I need not say a word now!
being, “Don't be late for the class
was when I had not joined Resonance and easy for a student to get lost in fun and when he is around. And outside the class, I tomorrow.” It was one memorable night.
was making up my mind for engineering. adventure. That way, he loses track of time believe, it is a privilege that I could rely on Future Planning? Have you anything
But a few tests, scorecards and enormous and progress. It is good to have friends, but him for anything I wanted. specific in your mind?
Details of your interested fields,
support from the teachers said that one must choose them wisely. And, if I haven't planned anything yet. Perhaps it
accomplishments and achievements:
disappointment just didn't exist as an parents are responsible for their children, Tell us something about your family. is time to leave the crowd!
Plenty of interest fields: photography,
option. the reverse is equally true. You owe them a My mother is a Professor at the playing guitar and casio, creative writing,
lot. Just keep that in mind. Work hard and Government Girls' college at my debating, listening to music, chatting with Once again, congratulations, and we
Whom you wish to give credit of your you will rise fast. hometown. She heads the BOTANY friends, sports like hockey, boxing (I boxed wish you happiness & success in all your
success in IIT-JEE? department there. My father is a Project thrice and never won a bout!), soccer, future endeavours.
There is a little credit that goes to me for What changes did you find in yourself Manager in Areva T & D, a French cricket and many more. Thank you so much for your wishes.
my rank. There were times when I wanted after joining Resonance? o rg a n i z a t i o n p i o n e e r i n g n u c l e a r There are no commendable achievements
to give up, go back home and ride bikes Most of the changes were subject oriented,
with my friends, party late night. But there
were people in my life who wanted me to
stay. They convinced me that it was the
best for me to keep going on. My parents
like a better understanding of general
Physics and Chemistry. I started fearing
Trigonometry the day I saw 'theta.' It took
me some time to throw that fear out. PT, CT
A Fair Share of Faith happy, smiling and tension free, regardless
of the circumstances they face. They
should partner their children like friends
and more important than this, they must
and my elder brother were there to back and JPT helped me in coping with the Hailing from a rural township in Madhya came to know about Resonance through have sustained faith in the institute and
me up for troubles on various scales, examination fear. I still remember the Pradesh, I come from a well educated newspapers, and we heard a lot of good their child's efforts.
cheering me up every time (sometimes uneasy cold bites I used to feel initially. family. Each of my family members sets his words about it through our friends, too. And for our son Archit, a greater part of the
when I didn't need it too). There were Overcoming them was another essential own objectives and targets, and then The entire journey of preparation for IIT- journey is yet to come!
teachers from Resonance and my school barrier. sets forth to achieve them. I passed JEE has been memorable one, towards the
especially Mrs. P. Jyothi who never let the through a hard life, full of struggles, end of which we witnessed the most
pressure seep in, and some good old Give some tips to increase efficiency though the desire to perform well never unforgettable moments of our lives. It is
friends who were there to inspire me with while studying. died. The educational life has been quite the Felicitation Function (VICTORY 2011)
their lectures, over a candy or a sizzling Efficiency is a very personal issue. successful, marked with good academic at Resonance in which we were honored as
paratha! Wow! Although, short term goals should be clear. records. Saying any amount of words distinguished guests as our child had
Very often, people sit on their study tables about IITs is less. Indian Institutes of qualified IIT-JEE 2011 with 14th All India Shri M.K. Gupta
What inspired you to plunge headlong wondering about the latest movies or Technology (IITs) are India's most Rank in general category. Archit's Father
into the preparation for Indian magazine covers. Sometimes, gossip prestigious & well recognized institutes for As far as parents are concerned, my
Institute of Technology (IITs)? sessions consume the entire night. With so engineering education. suggestion is that they should take enough
The crowd!!! Very often, facts are not so many friends around, it happens. But time, Resonance has a competitive, supportive care of their wards. Moreover it is
beautiful. I saw (and still watch it with joy) if lost once, cannot be brought back. One and engaging academic environment. We mandatory for parents to keep their kids

ANUNAAD - XXV (October-December 2011) 48 ANUNAAD - XXV (October-December 2011) 49


Umesh

Faces of Kulkarni
Saima Iqbal
Sarath
Babu
Dhruv

Young Anil Akkara


Lakra

Ajay Chaturvedi Anshu Jamsenpa

Nation Yuvraj Singh Sampa Arya


Prashant
Kumar
Dubey

Eight remarkable Young Indian HarVa is a unique and first of its kind Rural becoming the first woman (mother) old problems. Today, they have no drinking water Dubey who used RTI and the media
leaders who symbolise optimism BPO that focuses on skill development mountaineer in the world to reach the With the motto of empathy not charity, shortage, even in summers. Puzhakkal, the together to make child malnutrition one of
especially among rural women with an summit of Mt. Everest twice in the same Mirakle Courier employs a team 64 deaf river running through Adat, was for long the most important social, political and
of the country which has the
aim of creating jobs in rural India. Also it is season. The 32-year-old, mother of two, employees, where each of them has no better than a stinking drain. The electoral issues, fought for a change in
largest youth population in the
involved in reorganizing traditional from Arunachal Pradesh's Bomdila gained copious levels of confidence and is panchayat cleaned it up. The Puzhakkal order to bring transparency in our
world agriculture sector into community based successfully scaled the world's highest empowered with financial independence. River Tourism Village now attracts democratic system. Not only has he been
India has the second largest and youngest farming groups that are run by efficient peak to create a spectacular record with The social impact is that many of them are hundreds of tourists who come for a cruise phenomenal in engaging the Media in
population in the world. There are more and stable management system. grit and stamina, and became the first able to go back home and support their in the reborn river. And a rebirth it is for highlighting the issue of malnutrition
than 700 million Indians below the age of Founded in 2009, starting as a small BPO woman in the world to scale the mighty families, rather than being helplessly the village, too, under Akkara's able among children, but also been successful
35 in the country. Bharat Ratna Dr. Abdul venture in 2010 with just 50 women on its Mount Everest twice in one season in an dependent on them. leadership. in persuading the main opposition party to
Kalam once remarked that there was no first project in Tikli Akilampur village, a incredible span of just 9 days. It made a Saima Iqbal make child malnutrition an issue and take
Anil Akkara
credible platform in the country to few km from Gurgaon, Harva has so far huge financial burden for her middle class a stand on how the problem should be
The man responsible for reforming Adat With a degree from the United Kingdom,
recognize the achievements of youth trained over 500 women and hundreds of family but she was prepared to handle the addressed.
village with organic farming and sanitation Saima Iqbal is helping conserve Kashmir's
across various walks of life. farmers. It has centers in Haryana, terrifying prospects of failure. Yuvraj Singh (Special Award)
has become a role model for his village. rich cultural heritage. She has aided the
An inspired Young India means that India Rajasthan, UP and Uttrakhand employing Umesh Kulkarni Under his leadership for the last 10 years, mapping of Srinagar's cultural resources, Indian Cricketer and a member of the
is on an accelerated path to take its rightful hundreds of women. The company plans The 35 year old film maker is a National Adat village has received many prestigious worked on conservation of the 15th World Cup Winning Team, Yuvraj Singh
place in the world order as one of the to provide employment to 10,000 women Award Winner. His movie 'Valu' was awards like Swaraj Award for best century Aali Masjid and pushed Kashmir's was given the Special Award for
greatest nations in the world. and affect millions of lives across India by featured in Berlin film festival. 'Valu', also panchayat for eight years, the Central Mughal Gardens for the world heritage Outstanding Achievement, for his
Recently, IBN18 in association with IRB 2015. became first Marathi feature film to be proposal. performance in the recently concluded
government's Nirmal Gram Puraskar for
Infrastructure Developers Limited (IRB) Sarath Babu selected in Rotterdam International Film World Cup. Over the last couple of years,
achieving full sanitation and the Central The architecture of Jammu, Kashmir and
announced the winners for the 2nd Festival in the year 2008. His other film he has seen the depths of despair and
The man who turned into an Idli maker Water Resources Ministry's prize for Ladakh has always been as diverse as its
Edition of the 'Young Indian Leaders 'Vihir' was premiered at South Korea's experienced the summit of triumph.
after graduating from BITS Pilani and IIM, groundwater augmentation through cultural influences. Assaulted by apathy,
Awards' to honour the youth of the nation Pusan festival and also at the London film However, the tide turned at the World Cup
made his education the means to become rainwater harvesting. unchecked developmental pressure, and
for their admirable work and festival. His efforts are bringing back the where Yuvraj experienced the greatest
a businessman. Sarath Babu, now One of his key successes was ridding 3,000 vandalism, it has also suffered from two
commendable contributions. These young nations focus on Marathi cinema. moment of his career. Not only did he
synonymous with FoodKing, is an IIM- acres of paddy fields in the waterlogged decades of political instability, whether it
leaders are a shining example and a source perform superbly throughout the
Ahmedabad graduate who gave up Dhruv Lakra low lying lands of intense contamination is the arson of the Chrar-i-Sharief shrine or
of inspiration for the entire nation. They tournament but also played an
several high profile job offers to live an An MBA graduate from Oxford, Dhruv by shifting to organic farming. Until then the Tourist Reception Centre, Srinagar has
personify exemplary courage, strong will instrumental role in taking India to their
altogether different dream of providing started Mirakle Couriers, a courier these fields annually consumed 1.5 tonnes 838 listed properties, though only 18 are
power and a burning desire to overcome 2nd crown after nearly three decades.
employment opportunities to people. He company with a difference which employs of Furadan, a highly toxic pesticide that Grade 1 and enjoy legal protection. For
difficult circumstances to help the less
launched the Foodking Catering Service, only specially challenged (deaf & dumb) poisoned the nearby water bodies. Adat's Saima Iqbal, who graduated in Sampa Arya(Special Award)
fortunate ones. They come across from the
and now with the help of his younger youth thinking that a courier service domestic waste mixed with cowdung is architecture from MSIA, Bijapur, and 29 year old Sampa Arya from Sonepat
length and breadth of India, and have
brother and a team of 160 people he is requires minimal use of speech and converted into organic manure. A 20- joined INTACH in 2004, restoring the became the face of the campaign to 'save
worked for change in their own states as
looking to expand operations to cover the hearing. Barring the 4 management staff, member society of women under the state's cultural sanctity is a mission. With a our sailors '. Her husband Ravinder Singh
well as on a national level.
campuses of all major companies across all employees are hearing disabled. He has state-level women's empowerment degree from the UK, she has not only Gulia, 3 rd officer on Merchant Vessel Suez
Ajay Chaturvedi the country and abroad. He dreams to 2 branches in Mumbai and already plans project, Kudumbasree, supervises the helped in mapping Srinagar's culturaI along with 5 other Indian sailors were
Social entrepreneur and a BITSian of 1995 witness a hunger free India thirty years to go pan India in the long run. project. These women from BPL families resources by boldly walking its bylanes but taken captive in August 2010. Sampa led
batch, Mr Ajay Chaturvedi who is the from now and also stresses upon every kid While the award brings in much joy, make about Rs 5,000 every month by also worked on the conservation of the the campaign & knocked the doors of
Founder and Chairman of the company being computer savvy, being able to Mirakle Courier also inspired book author selling manure. Every home in Adat has 15th century Aali Masjid and the World Union Ministers, Chief Minsiters, even the
Harva, was awarded the CNN IBN Youth speak, read and write English in addition Rashmi Bansal, who has to her credit toilets. It is also the first panchayat to Heritage status proposal for Kashmir's six Prime Minister. Her fearless and relentless
Icon - Young Indian Leader of the Year to their mother tongue. He is a person who bestsellers like Stay Hungry Stay Foolish implement a housing scheme for all its Mughal Gardens. She is part of the team campaign forced the government to
Award for 2011. His company, Harva knows the hardships of people and who and Connect the Dots. Her latest book- I homeless and landless. The panchayat set working on the restoration of downtown debate the State response to the piracy
means Green for the villages and it stands can truly support them in every possible have a Dream, released in June 2011, up an iron treatment plant with Srinagar, which has faced the neglect of menace. Pakistani human rights activist
for Harnessing Value of rural India way and help them sustain their lives. features Mirakle Courier as one of the 20 contributions from the 3,500 beneficiary over 250 monuments within a 3-km area. Ansar Burney, moved by the plight of the
focusing on skill development, BPO, Anshu Jamsenpa inspiring stories of Social Entrepreneurs families and technical support from an Prashant Kumar Dubey sailors began negotiations for the release
Community Based farming and from India who found new ways to solve engineering college at a cost of Rs 2 crore. An RTI crusader from Bhopal, Prashant of the Suez crew that included Pakistani
She created a niche in her field by
Microfinance in rural India. nationals.

ANUNAAD - XXV (October-December 2011) 50 ANUNAAD - XXV (October-December 2011) 51


Jh cq)k dne dk jkgh

W
(A RELIGIOUS DEDICATION fn[kk x, gSa ftlus tyok
n`<+ bPNqd ty/kkjk lkA

O
TO RESONANCE) dne c<+k ihNs yksxksa dk
deZ izkS<+ eu I;kjk lkAA
lquks ,d ckj dh ckr iqjkuhA

B
eq>s ^^Jh cq)k** dh dFkk lqukuhAA izLrqr d:¡ dqN ,sls tyok
[kwc f[kyh 'kj.ku dh xkFkkA vfM+x fgEer dk L;kgh g¡wA
cpiu esa dbZ ckj lquk FkkAA cl vkxs gS pyrs jguk---
fcy[k & fcy[k cl jksos u;ukA

N
eSa dne dk jkgh g¡wAA
tc [kks, iq=] tu] cq) ds ;qxekAA
gj eaxyokj fi'kkp iwtk esaA

I
?kVdj ?kus ?kVk mrjdj
MY POEM IS DEDICATED TO nsr tu&cfynku ryokj Hkqtk esaAA ¼vaxqyheky½ ?kkV & ?kkV ij cksys gSaA
tc vR;kpkjh c<+r gh tkokaA
ALL RESONANCE xq: fi'kkp & iwtk mls 'kfDr fnykokaA
D;k leUnj D;k ;s vkleka
Hksn ;s lcds [kksys gSaAA

A
rc 'kfDr'kkyh og :i ys /kjghaA
xq: dk egRo dHkh gksxk uk de] 'kdr u jktk dqN Hkh u djghAA Lkh[k ldk rks f'k[kj fd Hkkafr
Hkys dj ysa fdruh Hkh mUufr ge] vR;kpkj ls gqbZ tc la[;k vk/khA iz[kj lh[k lEeku gqvkA
[kqyh vk¡[k cq) dh rksM+ lekf/kAA fxj iM+k rw mlh f'k[kj ls
oSls rks gS] Internet ij gj izdkj dk Kku] fudy iM+s ,d vL= u ysdjA (Illustration by Shiv Kant Singh)
tks FkksM+k vfHkeku gqvkAA
ij vPNs & cqjs dh ugha gS mls igpkuA ?kcjk, _f"k & eqfu Hkh cq) dks ns[kdjAA
R

ugha gSa] 'kCn dSls d:¡ /kU;okn] Lwfpr gqvk tc jktk dks i=A phj ld¡w /kkjk ufn;ksa dk
cl pkfg, gj iy vki lc dk vk'khZokn] ys & yks izHkq dksbZ ,d vL=AA /kkj eSa rks lqjkgh g¡wA
Mj&dj jg xbZ tx dh vfHkyk"kkA lwuh rhFkZ ;k dfBu jkg
g¡w tgk¡ vkt eSa] mlesa gSa cM+k ;ksxnku] ^^er tkvk vki** Fkh xk¡o fd Hkk"kkAA
vki lc dk ftUgksaus fn;k eq>s bruk KkuA eSa dne dk jkgh g¡wAA
,d u lquh tu & tu dh ckrsaA
vkius cuk;k gS] eq>s bl ;ksX;] ^^er ?kcjkvksa** cl ;gh fl[kkrsaAA ns[ku Hkyu~ vuns[kh esa
fd izkIr d:¡ eSa viuk y{;] eqLdqjkdj izHkq us dne c<+k;kA fudy iM+s dneksa ds ihNsA
fn;k gS] gj le; vkius lgkjk] ns[ks vc izHkq dh vije ek;kAA ekSu cus Nwus dks eafty
^^:d] Bgj!** lk dqN vkokt lh vkbZA jkse jkse esa fgEer lhpsAA
tc Hkh yxk eq>s fd eSa gkjkA oks ekSle Hkh dqN le> u ikbZAA
ij eSa g¡w fdruk erych] vaxqyheky Fkk lkeus vk;kA pØokr iM+s ik¡o tSls eSa
;kn fd;k uk vkidks dHkh] feyk utj vkSj dqN Fkk crk;kAA thr dk [kqn xokgh g¡wA
vkt djrk g¡w fny ls vki lc dk lEeku] [khapk ryokj Fkk E;ku ls iwjhA izK u;u Hkh cksy iM+s SHIV KANT SINGH
vki lc dks gSa] esjk lr&lr iz.kkeA vkSj fn[kk nh viuh vkSdkr v/kwjhAA eSa dne dk jkgh g¡wAA Roll No.: 1032334
bZ'oj dh d:.kk vlhe mi dk;kA
Batch: KTJPA-6
iy esa lc NksM+ dj pj.k esa vk;kAA
THIS POEM IS DEDICATED TO
MR. K.D. SIR (MATHS FACULTY) STEPS OF SUCCESS
izgjh cusa A Teacher is………
mDream is not what you see in sleep, dream is the thing which does not let your sleep.
mThe world we have created is a product of out thinking; it cannot be changed without changing our thinking.
Å¡ph & uhph tkr u gks Someone who is wise………….. mThe difference between 'impossible' and 'possible' lies in a persons' determination.
HksnHkko dh ckr u gks] Who cares, about the students and, wears no disguise mCourage is what it takes to stand up & speak; Courage is also what it takes to sit down and listen.
g¡l ds gj nq[k ih tk;s But is honest and open and share from the heart. mTalent is born in silence but character is born in the Struggle of Life.
vk¡lw dh cjlkr u gks]
eu esa lwjt&pk¡n jgsA Not just lessons from books, but from life, where you are, mIf you want to be a winner, hang around with winners.
xgjh &dkyh jkr u gks] A Teacher takes time to help a tutor, mYour treasure house is within; it contains everything you need.
vkyl & >wB & ykypksa ls With English or Maths or on a Computer mWhat does it take to be a champion? Desire, dedication determination, concentration and the will to win.
dHkh gekjh ekr u gks] mIn character, in manner, in style, in all things, the supreme excellence is simplicity.
It's K.D. Sir, who's patient, even in stress,
izgjh cus ns'k ds ge
dksbZ Hkhrj ?kkr u gksaA Who never gives less than the very best! mIt is through cooperation, rather than conflict that your greatest success will be derived.
Not that I was the perfect student, mIt's simply a matter of doing what you do best and not worrying about what the other fellow is going to do.
But you were the perfect teacher for me. mGreatest reward for doing is the opportunity to do better.
“A Good teacher is like a candle it consumes
It self to light the way for others.”
Shubham Kumar
Hemant Kumar Bajiya “What the teacher is, SIDDHARTH BAJIYA
Roll No.: 1003525
Roll No.: 11107777 Is more Important Roll No.: 11107775
Batch: KTPPH3
Batch: KIJAAH1 than what he teachers.” Batch: KTJAAH-1

ANUNAAD - XXV (October-December 2011) 52 ANUNAAD - XXV (October-December 2011) 53


Solutions of Physics - Reso Q. 23 Solutions of Maths - Reso Q. 23
Solutions by JKP Sir
Solutions by Safdar Seraj (SS Sir)
Sr. Reso Faculty Member, Physics HOD - Maths, Patna Centre

Q.1 Prove that the set of numbers {1, 2, ..... , 2005} can be colored with two colors such that any of its
18-term arithmetic sequences contains both colors.
Q.1 V1 = 400 m/sec. V2 = 1200 m/sec. the figure. Take only electrostatic interaction between
charges. Find: 2005 - a
Sol. The solution involves a counting argument that shows there are arithmetic
q1 = 4 mC q2 = – 10 mC (a) What will be their distance of maximum separation? 17
that the total number of colorings exceeds those that
m1 = 2 × 10–5 kg m2 = 4 × 10–5 kg (b) If V1 = 400 m/sec. and V2 = 2000 m/sec. What will be
m a ke s o m e 1 8 - t e r m a r i t h m e t i c s e q u e n c e squences that start with a. Altogether, the number of arithmetic
36 mm their distance of maximum separation? sequences does not exceed
monochromatic.
Two point like charges are moving in the same direction (c) In part (c), what will be their relative speed at the instant
There are 22005 colorings of a set with 2005 elements by 2005
(rightward) and corresponding numerical values are shown in of maximum separation?
two colors. The number of colorings that make a fixed
18-term sequence monochromatic is 22005–17, since the
 2005
a=1
17
-a =
2004 . 2005
2 . 17
1 1
Sol. (a) T. E. of system = × 2 × 10–5 × (400)2+ × 4 × As T.E. of system > K.E. of centre of mass terms not belonging to the sequence can be colored So the total number of colorings that makes
2 2
9 6 6 Hence particle separated to infinite distance. without restriction, while those in the sequence can be an arithmetic sequence monochromatic does
10–5 × 12002 – 9  10  4  10 3 10  10 r®¥ Ans. (a) colored either all black or all white. not exceed
36  10
(c) At r ® ¥ How many 18-term arithmetic sequences can be found 2004 - 2005
= 1.6 + 28.8 – 10 T.E. = K.E. of system 2 2005-17
in the set (1,2,....,2005) ? Such a sequence a, a + r, a + 34
= 20.4 J 2r, ......, a + 17r is completely determined by a and r which is considerably smaller than 22005.
1 1 1
K.E. of centre mass= (m1 + m2) Vcm2 m1V12 + m2V22 = 71.6 subject to the condition a + 17r £ 2005. For every a The conclusion follows.
2 2 2 2
1 2  10 5  400  4  10 5  1200
= ×6×10–5× = 26.13 J V12 + 2V22 = 71.6 × 105 ...........(i)
2 6  10 5
Conservation of momentum :
As K.E. of centre mass is greater than T.E. of system, they will not m1u1 + m2u2 = m1v1 + m2v2 Q.2 We are given the parabola y2 = 2px with focus F. For an integer n ³ 3 consider a regular polygon A1A2 ... An whose center is F
reach infinity. 2×10–5×400+4×10–5×2000=2×10–5V1+4×10–5 V2 and such that none of its vertices is on the x-axis. The half-lines |FA1,|FA2 ,... |FAn intersect the parabola at B1, B2, ... Bn. Prove
i.e. at r = ¥, U = 0 and T. E> of system will be equal to K.E. of that FB1 + FB2 + ... + FBn ³ np.
V1 + 2V2 = 4400 ...........(ii)
centre of mass.
On Solving : V2 = 1123.479 m/sec.
Hence, Kq q Sol. Set F Bk = tk, k = 1, 2,..., n. Also, let a be the angle made To prove this inequality, note that
1 2 V1 = 2153.042 m/sec.
T.E. = r + K.E. of centre of mass by the half-line |FB 1 with the x-axis and (1 – cos a1) + (1 – cos a2) + .....+ (1 – cos an)
max Vrel = V1 – V2 = 1029.563 m/sec.
 9  10 9  4  10  10 12 ak = a + 2(k  1) , k = 2, ....,n. The coordinates n
n 2(k  1)
20.4 =
rmax
 26 .13
P l e a s e a c c e p t o u r In the question V = 400 m/sec.
1 V = 1200 m/sec.
2
p
=n–  cos 
n
apologies for the of the focus F are ,0 . k 1
q = 4 mC 1 q = – 10 mC 2
rmax = 6.28 cm. Ans. (a) typographical error in m = 2 × 10 kg
1
–5
m = 4 × 10 kg2
–5
2 n n
Reso Q. 23.1 of Physics. 36 mm 2(k  1) 2(k  1)
(b) T.E. of system = 1.6 + 80 – 10
= 71.65 J
should have been typed as In general, the coordinates of the points on a ray that = n – cos   cos
k 1 n
 sin  sin
k 1
 n
 n.
originates in F and makes an angle b with the x axis are
V1 = 400 m/sec. V2 = 1200 m/sec.
K.E. of centre of mass = 64.53 J By the Cauchy - Schwarz inequality.
q2 = – 10 mC p
m1 = 2 × 10–5 kg m2 = 4 × 10–5 kg  t cos , t sin  , t > 0 (just draw a ray
36 mm
2 1 1 1
Q.2 An aeroplane is flying from a point A whose coordinates are (5 km, 2 km, 5 km).    
Consider two points B and C in the space whose coordinates are (- 2 km, 4 km, 0 km) from the origin of the coordinate system that makes an 1  cos 1 1  cos  2 1  cos  n
Consequently, the question became invalid. We
and (5 km, 1 km, 2 km). Assume that plane starts flying at t=0, in the plane of given have investigated the cause & to prevent re- angle b with the x-axis; then translate it to F). It follows n2 n2
three positions with speed 200 m/sec and acceleration 4 m/sec2 in a direction occurrences, we have modified our verification that the coordinates of Bk are    n.
process. Still, we appreciate the students who (1  cos 1 )  (1  cos  2 )       (1  cos  n ) n
perpendicular to line BC till it becomes collinear with line BC at time 't'. Find 't'. made an effort to attempt this question. Keep up p
 t k cos  k , t k sin  k k = 1, 2, ....,n. The equality case would imply that all ak's are equal, which is
Sol. (5, 1, 2) BA x BC = iˆ ˆj kˆ = 13 iˆ + 28 ˆj - 7 kˆ the good efforts! 2
d 90º C impossible. Hence the inequality is strict, as desired.
7 -2 5
Winner The condition that Bk belongs to the parabola is written
7 -3 1
as r2k sin2 ak = p2 + 2ptk cos ak. The positive root of this
(-2, 4, 0)
equation is tk = p / (1 - cos ak). We are supposed to prove
Winner
A | BA x BC | = 132 + 282 + ( -7)2 = 1002
(5, 2, 5) B that t1 + t2 + .... + tk > np, which translates to
r | BC | = 72 + ( -3)2 + 12 = 59
rA = 5 î + 2 ĵ + 5 k̂ 1 1 1
r 2     n.
rB = - 2 î + 4 ĵ + 0 k̂ 1002 d = 200t + 2t 1  cos 1 1  cos  2 1  cos  n
d= km, 2 - 100 ± 10 4 + 8240
rC = 5 î + ĵ + k̂ 59 2t + 200t - d = 0 t=
r r 2
2
BA = rA - rB = 7 î - 2 ĵ + 5 k̂
= 4.12 km 2t + 200t - 4120 = 0
r r Shahzma Alif Ravish Agrawal
t + 100 t - 2060 = 0 = 17.5 Sec
2
BC = rC - rB = 7 î - 3 ĵ + k̂ = 4120km Roll No.: 970251 Roll No.: 11104288
| BA x BC | Batch: A-2 Batch: RJ1
d=
|BC |

ANUNAAD - XXV (October-December 2011) 54 ANUNAAD - XXV (October-December 2011) 55


Solutions of Chemistry - Reso Q. 23
Solutions by S. K. Sinha (SHK Sir)
Sr. Reso Faculty Member, Chemistry

Q.1 All stereo-isomeric form of given compound x is treated with excess of Ph-NHNH2 (Phenyl hydrazine)in acidic medium. The
productsformedarefractionallydistillated.Ifmisthetotalnumberoffractionaldistillatesthemvalueofmis?
H3C O

O C H3

Sol. H3C N PHYSICS Sol 3. When connected in series, then


[RP + RQ] [1+aDT] = RP [1+aP DT] + RQ[1+aQ DT]
5
* Q   8  10  6  105 
FORCE
A * OH RP
= = = 1
HO RQ   P 6  10
5
 4  105
N C H3
When connected in parallel
Sol.1 2p
2p N RP RQ RP RQ [1   P T ][1   Q T ]
[1+a’DT]=
RP  R Q R P [1   P  T ]  R Q [1   Q T ]
H3C N
B * * OH 600 sin30º Neglecting product aP aQ :
o
30
RP R Q RP RQ [1   P T   Q T ]
N C H3 [1 + a’DT]=
600N RP  R Q R P [1   P T ]  R Q [1   Q T ]
y
OH
HO But RP = RQ
1  ' T 1   PT   QT
H3C N x =
Omdutt Choudhary fx = 0 2 2   PT   QT
C * *
Roll No.: 1019694 -2P - 2P cos 30º + 600 Sin 30º = 0 2+aPDT+aQDT+a’DT [2+aPDT+aQDT]=2+ 2aPDT+2aQDT
Batch: PH6 2P [1+Cos 30o] = 300 Neglecting a’ aP and a’aQ,then :
N C H3
2a’DT = aPDT + aQDT
300
OH P= N P   Q
3+2 a’ = = 6 × 10–5 /ºC.
For all A,B and C two structures ( cis and trans) at the marked stereo -center is possible. 2
fy = 0
Q.2 The products formed in the reaction sequence, from compound y, are fractionally distillated. If n is the total number of fractional N - 600 Cos 30º+2P Sin 30º=0
distillates them value of n is? Sol 4. For adiabatic process : Integrating both side
300
OH N= 300 3 DW = –DU T P
3+2 dT dP
O O OH 1. 
or ncvdT = – P.dv  T
= R 
[a  R ] P  b
O H CH3
2. fraction distillation n no of product
=
600 + 600 3
=
600 ( 3+1 ) N
But PV = nRT T0 P0
HO CH3 3+2 or P.dv = nRdT – V.dP T R ℓn b +[a +R ].P
3+2 ℓn =
Again, ncvdT = v.dp – nRdT To a+R b+[a+R] P0
DO
O Sol. 2
Sol. O ncvdT = nRT dP  nRdT T
a R
b  [a  R ] P
R
h = 1 gt2 …(1) 0.36h
t=() P or 
O 2 RT To b  [a  R ] P0
H CH3
[cv + R] dT = dP
P
O 0.36 h = 1 g (t - 1)2 …(2) h
CH3 2 b RT
t= (t-1) a   R dT  dP  Ta+R[b+(a+R)P]-R= constant.
2 P P
D  t=t
O H 1 dT R dP
O =
and 0.6 = t - 1 T [ [a + R ]P + b[
O t
H CH3
O Aditya Garg 0.6t = t-1  t = 2-5 Sec
CH3
Roll No.: 1005953
h = 1 gt2= 1 x10x 25 = 250 = 31.25 m
MATHS 1.(D) 2.(B) 3.(C) 4.(D)
6.(A) 7.(D) 8.(C) 9.(C)
5.(B)

O H D
Batch: PA1
2 2 4 8
EXPONENT
Only two diastereomeric products hence 2 fractional distillate.

ANUNAAD - XXV (October-December 2011) 56 ANUNAAD - XXV (October-December 2011) 57


O LYMPIAD
OLYMPIAD
Olympiad - Physics B=3
 i
4 R
(sin 1  sin  2 ) RES Winners'
Page
Sol.1 The magnitude of the magnetic field at the centroid 0 of
the triangle due to a side PQ (say) is
 i
4 R
(sin 1  sin  2, )
where R is the perpendicular distance of PQ from f1 and
Here i =1.0 A, f1=f2 =60º and R=(l/2) cot60º=l/2 3 ,
where l is the side of the triangle (=4.5 ×10–2m).

\ B = 3 × 10–7 × 2
1 .0 3


4.5  10 / 2 3  2


3

2 
REPERT IRE
f2, the angles as shown. The field is perpendicular to the
plane of page, directed downward. Since the magnetic 3  10 7  2  3
= = 4.0 × 10–5 T.
2
field due to each of the three sides is the same in 4.5  10
magnitude and direction, the magnitude of the resultant Sol.2 Let the desired temperature rise be DT. Hence, the
field at 0 is. diameter (linear dimension) of the brass rod d'b is given
by d'b= db (1 + ab DT) ------ (i)
Similarly, the diameter of the steel rod d's is given by
d's= ds (1 + as DT) ------ (ii)
For the steel ring to fit over the rod, d's =d'b
or ds (1 + as DT) = db (1 + ab DT)
ds – db
or DT =
dbb – ds  s Ravish Agarwal Shahzma Alif
Roll No.: 11104288 Roll No.: 970251
Batch: RJ1 Batch: A2
Winner of Maths Reso Q. 23 Winner of Physics Reso Q. 23

Olympiad - Chemistry
Sol.1 Na2SO3 + S  Na 2(SA )2O 3 Sol.2 (COOH)2 Conc . H2SO4 H2 O + CO2 + CO
(D )
(A) (B ) (C )
Neutral Acidic Diatomic
oxide
Na2S2O3 + 2AgNO3  2NaNO3 + Ag2 S2 O3 COCl2 NH3 NH2 CONH2
(B ) CO + Cl2
Ag2S2O3 + H2O (E) (F)
 Ag2 S + H2SO4 Poisonous
(C )

Olympiad - Maths Omdutt Choudhary Aditya Garg


Roll No.: 1019694 Roll No.: 1005953
Sol.1 m , n are integer Sol.2 Let AB = a then BE = a tanq Batch: PH6 Batch: PA1
1/3 < m/n < 1 ...(i) Winner of Chemistry Reso Q. 23 Winner of Chemistry Reso Q. 23
CE
= tanq
m   = m and n ¹ 0 CF
n n
CF = a cotq – a
m Dear students! Wear your thinking caps! Your question can
a = m  = +ve integer GH = FD
1 win you a prize!!
= a – CF All the students are invited to contribute brainy questions in Physics,
\ m=2 = a – (a cotq – a) Chemistry & Maths pertaining to 10+2 syllabus of NCERT. Questions should
Using equation (i) = a (2 – cotq ) Now, In DGHF be original. The best question will be selected by the faculty members of

tanq = HF = 1  tan 
Resonance and will be published in the next edition of 'ANUNAAD'. The
n = 3, 4, 5 In D ABG
GH 2  cot  contributor will also fetch a prize for the best question.
m
\ = 2 , 2, 2 AG AG It will be a contribution by the students for the students and will surely infuse
n 3 4 5 tan q = tan q = a Solving we get an oomph in the students across the country to cultivate their mental
AB faculties.
2 2 Criteria for selecting the best question:
AG = a tan q tanq = \ sinq =
3 13 • Question should not be copied from any where.
HF = DG = a – a tan q = a ( 1 – tan q ) • Question should not be similar to any existing question.
• Question should be along with complete solution.
• Question should be conceptual, may not be tough.
ANUNAAD - XXV (October-December 2011) 58 Submit your question at J2, Help Desk or e-mail at editor@resonance.ac.in or Fax at 0744-2427144

Vous aimerez peut-être aussi